Sei sulla pagina 1di 97

Table of Contents

CHAPTER 1 THE LAW AND ETHICS OF LAWYERING.........................6


Disclosure of Adverse Evidence............................................................6
Spaulding v. Zimmerman et al.............................................................................6
Evans & Luptak, PLC v. Lizza et al........................................................................7
Virzi v Gran Trunk Warehouse & Cold Storage Co.................................................8
Brown v County of Genesee.................................................................................9
People v Belge...................................................................................................... 9

CHAPTER 4 CONFIDENTIALITY....................................................11
Confidentiality...................................................................................11

Privilege..................................................................................... 11
Aspects of the Privilege.....................................................................12
Competing Views on the Application of Privilege Wigmore / Upjohn................12
Exceptions to Attorney-Client Privilege 81-85 Restatement of Law Governing
Lawyers.............................................................................................................. 13
Corporations and the Attorney-Client Privilege....................................13
Upjohn v. United States (Supreme Court)...........................................................13
Documents, Attorney-Client Privilege and the Bill of Rights.................14
Fisher v. United States........................................................................................ 14
United States v. Hubbell..................................................................................... 15
The Crime-Fraud Exception.................................................................15
United States v. Chen......................................................................................... 15
Waiver of Attorney-Client Privilege.....................................................16
In re Columbia/HCA Healthcare Corp. Billing Practices Litigation.......................16
Transactions with Persons Other than Clients......................................17

Professional Duty of Confidentiality.............................................18


Scope of Duty of Confidentiality.........................................................18
Confidentiality and Joint Clients..........................................................18
Overview of Exceptions to Confidentiality...........................................19
Self-Defense Exception...................................................................................... 19
- Meyerhofer v. Empire Fire and Marine Ins. Co.................................................20
Client Fraud........................................................................................................ 20
- O.P.M.: A Case Study and its Lessons..............................................................20
Confidentiality when Death or Bodily Harm May Result.....................................21
- Macumber....................................................................................................... 21

CHAPTER 5 CONFLICTS OF INTEREST..........................................23


Purposes, Origin and Scope of Conflicts of Interest..............................23
Eternalness of Confidentiality.............................................................24
Concurrent Representation in Litigation..............................................24
Concurrent Representation in Civil Litigation......................................25
1

Westinghouse Elec. Corp. v. Kerr-McGee Corp....................................................25


Fiandaca v. Cunningham.................................................................................... 26
Curing a Simultaneous Conflict: The Hot Potato Doctrine vs. Thrust-Upon
Conflict...............................................................................................27
Remedies for Conflicts of Interest Violations.......................................28
Concurrent Representation in Criminal Litigation................................28
Cuyler v. Sullivan................................................................................................ 28
Mickens v. Taylor................................................................................................ 29
Joint Representation in Transactions and Civil Litigation......................29
Representing Both Parties to a Real Estate Transaction.......................30
State v. Callahan Discipline Case.....................................................................30
Joint Representation in the Formation of a Business............................31
Successive Representation.................................................................31
Successive Representation of Joint Clients..........................................32
Brennans Inc. v. Brennans Restaurants, Inc.....................................................32
Taint Shopping and Duties to Prospective Clients................................34
Imputed Disqualification and the Migratory Lawyer.............................34
Nemours Foundation v. Gilbane.........................................................................34
Conflicts when Law Firms Merge.........................................................35
Remedies against a lawyer in conflicts of interest cases:.....................36
Impacts on a client where there is a conflict of interest:......................36

CHAPTER 6 WHO IS THE CLIENT?................................................37


Individual or Enterprise......................................................................37
Meehan v Hops................................................................................................... 37
EF Hutton & Co Brown........................................................................................ 37
The Organization and its Constituents................................................37
In Re Grand Jury Subpoena................................................................................37
Almost Clients...................................................................................40
Representing a Closely Held Corporation or Partnership......................40
Fassihi v. Sommers, Schwartz, Silver, Shwartz and Tyler, P.C.............................40
Representing a Fiduciary...................................................................41
Fickett v. Superior Court of Pima County............................................................41
Attorney-Client Privilege for a Trustee................................................................42
Hazard: Triangular Lawyer Relationships........................................................42
Three Possible Relationships.............................................................................. 43

CHAPTER 7 LITIGATION..............................................................44
Client Perjury.............................................................................44
Perjury in Civil Cases.........................................................................45
United States v. Shaffer Equipment Co..............................................................45
Perjury in Criminal Cases...................................................................46
Nix v. Whiteside.................................................................................................. 46
Monroe H. Freedman Perjury: The Lawyers Trilemma.......................49
2

United States v. Gellene..................................................................................... 49


Obstruction of Justice and the Anderson Case...................................................51
Conflict of Interest Hypo: Lawyer as a Witness....................................51

CHAPTER 8 LAWYER-CLIENT RELATIONSHIP................................53


Fees........................................................................................... 53
Amount of the Fee.............................................................................54
In Re Fordham.................................................................................................... 54
Bushman v. State Bar of California.....................................................................56
Judicial Control of Fees.......................................................................57
Non-Refundable Retainer Fees...........................................................57
Illegal Fees........................................................................................57
Fraudulent Billing and Fee Padding.....................................................57
Fee Disputes.....................................................................................57
Fee Arbitration..................................................................................57
Client Confidentiality in Fee Disputes..................................................57
Contingent Fees................................................................................58

Lawyer-Client Transactions..........................................................59
Transactions with Clients...................................................................59
Business Transactions........................................................................59
Committee On Professional Ethics and Conduct of Iowa State Bar Association v.
Mershon............................................................................................................. 59
Lawyer Equity Investments in Clients..................................................60
Passante............................................................................................................. 60
Contracting with a Client for Rights to the Clients Story.....................60
Gifts from Clients...............................................................................60
Sexual Relationship with Client..........................................................60
Handling Property of Clients and Others.............................................61

Scope of Lawyers Authority........................................................62


Actual and Apparent Authority...........................................................62
Allocation of Decision-Making Authority in Civil Litigation....................63
International Telemeter Corp. v. Teleprompter Corp...........................................63

Ending the Relationship..............................................................64


Lawyer Withdrawal............................................................................64
Discharge and Its Consequences.........................................................65
Ending the Relationship Before it Starts..............................................65
Togstad v. Vesely, Otto, Miller & Keefe...............................................................65

CHAPTER 2 CONFORMITY TO THE LAW......... Pages 57 76; 93 118;


67
Criminal Law - Crimes Related to False Testimony................................67

Tort Law Claims by Non-Clients....................................................67


3

Negligent Misrepresentation..............................................................67
Greycas, Inc. v. Proud......................................................................................... 67
Professional Malpractice....................................................................68
Tort of Malpractice is made up of the following elements:....................68
Fraud and the Recklessness Standard of Intent...................................68
Assisting a Client in Tortious or Illegal Conduct...................................69
Hazard: How Far May a Layer Go in Assisting a Client in Unlawful Conduct?. 69

Chapter 7: Litigation (Part II).......................................................69


Frivolity, Harassment and Delay: Professional Rules and Attitudes.......69
Rule 11 Sanctions..............................................................................70
Affirmative Duty to Investigate...........................................................71
Other Procedural Sanctions................................................................71
Sanctions for Discovery Abuse............................................................72
Qualcomm v. Broadcom..................................................................................... 72
The Anders Brief................................................................................................. 72

CHAPTER 9 COMPETENCE..........................................................73
Ethics Rules on Competence...............................................................73
Malpractice.......................................................................................73
Overview of the Tort of Malpractice....................................................73
Standard of Care and Causation.........................................................74
Lucas v. Hamm................................................................................................... 74
Smith v. Lewis.................................................................................................... 74
Effective Assistance of Counsel Under the Sixth Amendment................75
Ineffective Assistance: The Constitutional Standard............................76
Strickland v. Washington.................................................................................... 76
Malpractice Claims Against Criminal Defense Lawyers.........................77
In re AMB, 248 Mich App 144 (2002)..................................................................77

CHAPTER 10 LAW, LAWYERS AND JUSTICE...................................78


Advertising and Solicitation...............................................................78
Personalized Letters to persons thought to have legal problems:.........79
Shapero v. Kentucky Bar Association..................................................................79
Written Solicitation of Accident Victims:.............................................80
Florida Bar v. Went For It, Inc............................................................................. 80
Free Speech Rights of Lawyers...........................................................80
Gentile v. State Bar of Nevada...........................................................................80

CHAPTER 11 THE STRUCTURE OF LEGAL PRACTICE......................81


Character and Fitness........................................................................81
In re Hale............................................................................................................ 81
In re Himmel....................................................................................................... 82
Obligation to Report Professional Misconduct of Other Lawyers...........82
Gambro.............................................................................................................. 83

Hypotheticals.............................................................................84

Exam: Purgery Hypo

CHAPTER 1 THE LAW AND ETHICS OF LAWYERING


Introduction
- Four problems with lawyering:
o Prohibited assistance: What kinds of things is a lawyer prohibited from
doing for a client?
o Competence: What measures will assure competent lawyering?
o Confidentiality: What information learned by a lawyer should she treat as
secret, and from whom, and under what conditions may the secrecy be lifted?
o Conflicts of Interest: When and to what extent is a lawyer prohibited from
acting because there is a conflict of interest between her clients or between
herself and a client?
Disclosure of Adverse Evidence
Spaulding v. Zimmerman et al.
Facts
Father brings action for son (minor at the age of 20) who was in a car accident.
Zimmerman was driver of the vehicle which was in an accident, and David
Spaulding (son) was a passenger in the vehicle. David suffered from Aorta
Aneurysm, likely because of the accident but his doctors were unaware of this.
The Defendants doctors determined that this existed but never revealed the
information to David, therefore David and father agreed to settlement claim, with
the thought that David was recovering well from accident.
- Concealment of knowledge is not being discussed
Issue

Can the settlement be vacated?


- Is there a causal relationship between the accident and the aneurysm?

Holding
Ratio

Settlement can be vacated


Except for special circumstances (such as mutual mistake, fraud on the court or
concealment from the court), courts will not set aside a judgment because a
lawyer has concealed adverse evidence from the opposing party. This case was
one of those special circumstances he was a minor and there was concealment
(1- between doctors, 2- bw doctor and patient, 3- bw D attorney and clients)
- Under Rule 60.02(6) of the R of CP ct was justified in vacating settlement
even though no legal obligation may have rested on D to disclose
- Court may vacate such a settlement for mistake even though mistake
was not mutual
o Must be concealment or knowledge by one party that the other
party is labouring under a mistake in order to set aside a release
for mutual mistake
o Equity prevents an unconscionable advantage
- While there was no canon of ethics or legal obligation to disclose,
it was obvious that the settlement made did not
contemplate/consider the disability described

Reason
ing

Keller v. Wolf: equity will prevent one party from taking an unconscionable
advantage of anothers mistake for the purpose of enriching himself at the

Notes

others expense
Rule 60.02(6) of Rules of Civil Procedure

Multiple concealments here

Model Rule 1.4 Duty of Communication to a Client


(a) A lawyer shall:
(1) promptly inform the client of any decision or circumstance with respect to which the client's
informed consent, as defined in Rule 1.0(e), is required by these Rules;
(2) reasonably consult with the client about the means by which the clients
objectives are to be accomplished;
(3) keep the client reasonably informed about the status of the matter;
(4) promptly comply with reasonable requests for information; and
(5) consult with the client about any relevant limitation on the lawyers conduct when
the lawyer knows that the client expects assistance not permitted by the Rules of
Professional Conduct or other law
(b) A lawyer shall explain in a matter to the extent reasonably necessary to permit the
client to make informed decisions regarding the representation
Model Rule 1.6 Confidentiality of Information
(a) A lawyer shall not reveal information relating to the representation of a client unless the
client gives informed consent, the disclosure is impliedly authorized in order to carry out the
representation or the disclosure is permitted by paragraph (b).
(b) A lawyer may reveal information relating to the representation of a client to the extent
the lawyer reasonably believes necessary:
(1) to prevent reasonably certain death or substantial bodily harm;
(2) to prevent the client from committing a crime or fraud that is reasonably certain to result
in substantial injury to the financial interests or property of another and in furtherance of
which the client has used or is using the lawyer's services;
(3) to prevent, mitigate or rectify substantial injury to the financial interests or property of
another that is reasonably certain to result or has resulted from the client's commission of a
crime or fraud in furtherance of which the client has used the lawyer's services;
(4) to secure legal advice about the lawyer's compliance with these Rules;
(5) to establish a claim or defense on behalf of the lawyer in a controversy between the
lawyer and the client, to establish a defense to a criminal charge or civil claim against the
lawyer based upon conduct in which the client was involved, or to respond to allegations in
any proceeding concerning the lawyer's representation of the client;
(6) to comply with other law or a court order; or
(7) to detect and resolve conflicts of interest arising from the lawyers change of
employment or from changes in the composition or ownership of a firm, but only if the
revealed information would not compromise the attorney-client privilege or otherwise
prejudice the client.
(c) A lawyer shall make reasonable efforts to prevent the inadvertent or unauthorized

disclosure of, or unauthorized access to, information relating to the representation of a


client.
Rule 1.6
- Relates to a lawyers ability to disclose information regarding a represented client
- Contributes to the trust that is the hallmark of the lawyer-client relationship
o Confidentiality relates to all matter communicated in confidence by the client
as well as information relating to the representation, whatever its source
- There are exceptions to confidentiality as discussed in (b)
Evans & Luptak, PLC v. Lizza et al.
Facts
- This case involves an attorney conflict of interest. Plaintiff requests this
Court to enforce an unethical referral fee contract
-

Issue
Ratio
Reason
ing

Note

Car accident killed Catherine Stephensons husband (Robert), son (Brett),


and mother (Florence Budge)
Catherine was personal representative for both husband and son, both of
which were represented by same company Evans & Luptak
Was referred to file lawsuit for son against husband under wrongful death,
whereby defendant (Catherine) agreed to pay plaintiff (company) a onethird contingent fee for all recoveries obtained against plaintiffs client
o There was a conflict of interest within Evans & Luptak
As I advised you, yes, I believe it is a conflict of interest to
represent one party against another and then turn around
and represent the interests of the party against whom you
had already made a claim
Evans & Luptak backed away but new company still went after wrongful
death suit

Can Evans & Luptak seek a referral fee?


Law firm that referred case against client cannot recover a referral fee
- It is a well-established ethical principle that an attorney owes undivided
allegiance to a client and usually may not represent parties on both sides
of a dispute.
- A lawyer should not be allowed to sue an individual client on behalf of
another present client, even if the lawyer represents the first client in a
wholly unrelated matter, such as drafting his will. This follows, because
the focus of Rule 1.7(a) is on impairment of the client-lawyer relationship,
and it is unreasonable to postulate trusting relationships under those
conditions.
Remember: your authority ends at the time the client passes so you are obligated
to tell the court and opposition when this happens

Virzi v Gran Trunk Warehouse & Cold Storage Co


US District Ct for Eastern Dist. Of MI
Facts
- Defendants filed a motion to vacate a settlement order that confirmed a
settlement between plaintiff and defendants.
- Plaintiff was expected to be a good witness in his personal injury action.
- Plaintiff's attorney was unaware of his death at the time that a mediation panel
recommended a settlement. Several days later plaintiff's attorney was
informed of the death, and did not inform the court or opposing counsel.

Issue

Holdin
g
Reason
ing

Notes

Later, the parties settled for the recommended amount, the court placed the
settlement on the record, but plaintiff's counsel did not inform anyone of his
client's death when the settlement was confirmed and made part of the record.
Ps attorney claims his actions were not unethical/improper P was alive at the
time mediation statement was filed and there was nothing false/misleading in
the statement he did not become aware of the death until 3 days after award
of mediation panel
Ps attorney argues ct had the authority to enter the settlement order since a
personal representative had been appointed for P by probate court
Did Ps attorney have an ethical duty to advise the Court and Ds
attorney, who was unaware of the death of P, that P had died a few
weeks prior to the settlement agreement?
Ds claim settlement is void because Ds attorney did not know of Ps death at
time of settlement and Ps attorney failed to disclose
While the death was not caused by injuries related to the lawsuit and did not
have any effect on the fairness of the $35K mediation award, it had a
significant bearing on Ds willingness to settle
By not informing the Court of Ps death or not filing a motion to substitute
parties, Ps attorney led the Court to enter an order of settlement for a nonexistent party
Ps attorney owed a duty of candor and frankness to the Court and
such duty required disclosure both to the court and opposing counsel
of a significant fact such as death
- Opposing counsel does not have to deal with his adversary as he would
deal in the marketplace. Standards of ethics require greater honesty,
greater candor, and greater disclosure, even though it might not be in the
interest of the client or his estate.
Standards of ethics establish greater candour and honesty
Lawyer/Professional ethics = client over all others
However, the preference of ones dead client is not right
Unlike Spaulding, however, the holding of this case rests on a concealment
from the court rather than the failure to disclose to an adverse party

Brown v County of Genesee


US Ct of Appeals 6th Cir, 1989
Facts
- P was hired as county jail guard on condition she passed physical test
- She had diabetes and was not hired
- Lawsuit finally hired two years later
- P signed a stipulation that she be paid at the C step (bc P believed that
to be the highest step she could have received had she been hired on
June 16, 1982)
- Prior to approval of Ps stipulation, D was made aware that she could
have been paid at the D step
- D did not inquire if P was aware
- P became aware and brought motion to modify settlement agreement to
which she entered into with county allegedly under mistaken impression
regarding highest pay level to which she would have been entitled to
had she been hired immediately
- District Ct granted motion - concluded that the settlement agreement
should be vacated and reformed modified so P could be paid at D
step
- Employer appealed
Issue
- Was county responsible of advising the plaintiff of the potential to
receive a higher pay scale in the settlement?

Ct of Appeals: (1) There was no mutual mistake or fraudulent


misrepresentation warranting relief from settlement agreement; (2)
There was no fraudulent/unethical conduct by county warranting relief
from settlement agreement
Ct must enforce the settlement as agreed to by the parties and is not
permitted to alter the terms of this agreement (absent special
circumstances such as a material breach or duress)
District Ct abused its discretion by making factual conclusions
There was proper offer and acceptance of the agreement; There was a
meeting of the minds
MUST be fraud or mutual mistake to modify NOT a unilateral
mistake
With a unilateral mistake, not sufficient to allow the mistaken
party to avoid the effect of an otherwise valid settlement
There was no fraudulent or unethical conduct by the company

People v Belge

FACTS

Ratio
Reaso
n

County Ct of NY, 1975


Dead bodies case
D assigned as counsel for a man accused for murder (Garrow)
Spoke to lawyers and told them about the murder he is being charged
with and about 2 others he admitted lawyers didnt believe him.
Garrow then draws maps to the two bodies. Lawyers find the body in
cemetery and take pictures and do same for body in mine shaft. Keep in
mind that parents of these two women are making pleas for information
on their daughters whereabouts.
The discovery was not disclosed to authorities, but became public during
the course of trial when a plea of insanity was interposed on clients
behalf
D was thereafter indicted by the Grand Jury for violations of the Public
Health Law requiring that a decent burial be accorded to the dead, and
that anyone knowing of the death of a person without medical
attendance report it
Upon a motion by D for dismissal of the indictment on the ground for
confidential, privileged communication existed bw himself and his client,
which excused him from making full disclosures to authorities
The motion is granted on the grounds of a privileged communication and
in the interests of justice
Our system of criminal justice is an adversary system and the interests
of the State are not absolute, or even paramount
The effectiveness of counsel is only as great as the confidentiality of the
client-attorney relationship if the lawyer cannot get the full facts of the
case, he can only give his client half of a defense
The individual is protected by the Constitution by the right to counsel,
trial by jury, due process, privilege against self-incrimination
The clients 5th Amendment rights cannot be violated by his attorney
State Bar Association gets involved and writes that lawyers COULD NOT
release the information. Their ethical duty as lawyers was to not release
the information.
o You are supposed to talk FOR your client not ABOUT your client

10

So what is the duty of confidentiality?


Ultimately, ruled that if these guys had been prosecuted for
obstruction of justice then maybe you would have a case, but
not health code provision

11

CHAPTER 4 CONFIDENTIALITY
Distinction between confidentiality and privilege:
Confidentiality
- Confidentiality is a duty of Professional Conduct. It is governed by Rule 1.6.
o Information gained in the course of the representation
o This rule applies outside of the setting of the proceeding
o The Professional rule/duty of confidentiality is broader than the law of
attorney-client privilege
- A lawyers duty is to talk for a client, not about a client
- This is what applies when the lawyer is at the office, in public, etc.
- Does not require a proceeding
- Can continue after a proceeding
- Keeps a lawyers mouth shut about a clients affairs in everyday life
- Extremely broad
o The ABAs version of its definition of confidentiality reflects this
Rule 1.6: A lawyer shall not reveal information relating to the
representation of the client unless the client gives consent
- Talking about who your client is is confidential
Privilege
- An evidentiary rule that says (Wigmore on evidence) the privilege is what a client
tells the lawyer, when they know they are talking to their lawyer, for the purpose of
getting advice
- Also refers to the lawyer giving the advice to the client
- Requires a proceeding in order to be brought to life (because this is where
the rules of evidence apply)
- Allows a lawyer or client to refuse to give testimony on certain matters in court or in
another proceeding in which the evidence rules apply
- Generally info about who your client is (physical description, etc.) is non-privilege
o ABA Rule 1.9(c) a lawyer shall not use info pertaining to a former client
unless the information has become generally known
o Note the wording: generally, not publicly thus, even if a case is public, it
does not mean that it is generally known
o This promotes the confidential relationship a lawyer is selling to his/her
client the ability to hold secrets
- Ex of when a clients identity is in fact privilege:
o Where it is inextricably related to the incident
o i.e. footnote 20 on pg. 263 the lawyer represents the person accused of
leaving the scene of an accident. He will not provide the identity of that
person but he wants to make restitution on that persons behalf (wants the
client to meet the obligation to report without revealing who the client is
would create a larger issue by failing to report)
o i.e. it is a crime to fail to file your income taxes in the US. It is not uncommon
for someone to decide later on to anonymously pay your taxes. Hire a lawyer
to go to the IRS. A judge should not command the identity because in this
rare circumstance (either by case law or logic) that privilege would be
maintained
-

A privilege is another kind of a legal right, but one that usually takes the form of an
exemption from the duty to do something

12

Defined by the Restatement of Law Governing Lawyers as


o a communication
o made between privileged persons
o in confidence
o for the purpose of obtaining or providing legal assistance for the client
CONFIDENCE that you cannot be forced to expose to anyone else
Most sellable quality of a lawyer: ABILITY TO KEEP SECRETS

Aspects of the Privilege


- The rule only protects privacy in the context of encouraging full disclosure
- Privileged information extends only so far as the client is seeking legal advice from
a person whom the client reasonably believes is a lawyer (payment for advice is not
necessary)
- Termination of attorney-client relationship does NOT end the privilege; protection
continues even after clients death
- If a lawyer witnesses something or knows about some from some other source other
than client communication, the lawyer can be forced to answer did x happen?; but a
client cannot be forced to describe what he told his lawyer and a lawyer cannot be
forced to reveal that which a client told him in confidence
Underlying facts or evidence are not protected by the privilege, but ONLY a clients
communications to a lawyer made for the purpose of obtaining legal advice is privileged
- Presence of a third person
o Privilege is lost if it is not made in secrecy (presence of a third person destroys
privilege)
o Client waives privilege if, after communicating with a lawyer, the client tells
third parties what she said to the lawyer and what the lawyer said to her
- Client identity is not generally protected (it is generally a fact)
o Exception: if the client forms the last link in a chain of evidence (this is then
a question of confidence and not privilege)
o Protects only communications that are intended to be confidential thus,
physical characteristics of a client are not privileged
- Joint clients and Cooperating parties
o If two or more persons jointly retain a lawyer to represent them in a matter,
communications made by any of the clients to the lawyer on the subject of
joint representation are privileged against the rest of the world, but NOT
privileged against use by one joint client against another
- Whereabouts
o GENERALLY SPEAKING, whereabouts (if you happen to know them and they
were not given in a conversation of giving legal advice) are not privilege
o IF your client tells you where they are, seeking advice, this is generally
privilege
EXAM: PRIVILEGE OR CONFIDENTIALITY
- PRIVILEGE REQUIRES A PROCEEDING; IF NO JUDGE IT IS NOT PRIVILEGE
Model Rule 3.4(c)
- A lawyer cannot knowingly disobey an obligation under the tribunal except an open
refusal that no duty exists
Competing Views on the Application of Privilege Wigmore / Upjohn
- Wigmore
- 8 factor test for communications made in confidence

13

o (1) where legal advice of any kind is sought


o (2) from a professional legal adviser in his capacity as such,
o (3) the communications relating to that purpose,
o (4) made in confidence
o (5) by the client,
o (6) are at his instance permanently protected
o (7) from disclosure by himself or by the legal adviser
o (8) except the protection to be waived
- 2 common law factors
o Evidentiary privileges stand in derogation of the publics right to every
mans evidence
o Burden of establishing the elements of the privilege rests on the party
asserting it
Upjohn
- Applies a more liberal interpretation of the attorney client privilege
- Protects the downward flow of advice from lawyers and the upward flow of
information from those working at lower levels within the company
- The control group test may lead to a zone of silence within corporate affairs
which is why it was rejected in this ruling
- Two purposes behind the privilege:
- To preserve the observance of the law (telling clients what to do)
- To promote legal advocacy on behalf of the client (telling lawyers about the
situation)

Exceptions to Attorney-Client Privilege 81-85 Restatement of Law Governing


Lawyers
1. A dispute concerning a decedents disposition of property
2. Client crime or fraud
3. Lawyer self-protection
4. Disputes in which a trustee or other fiduciary is charged with a breach of fiduciary
duty by a beneficiary
5. Disputes between representatives of an organizational client and constituents of the
organization
Corporations and the Attorney-Client Privilege
Upjohn v. United States (Supreme Court)
Facts
Upjohn manufactures and sells pharmaceuticals. During audit, it was discovered
that Upjohn made payments to foreign governments to secure govt business
(many of which were illegal). Company sent requests and questionnaires through
general counsel to gather further information on this matter.
IRS later issued a summons demanding production of information collected by
questionnaires but Upjohn declined to produce them on the grounds that the
information was privileged as it was collected in preparation for litigation
purposes (privilege and attorney work-product doctrine)
Issue
Does the privilege extend to the company as a whole? (or just those employees
in the control group?
Holding Information collected by attorney in questionnaires and interviews IS privileged
information
- Rejection of control group test (privilege just applies to the control group
or the executives of a corporation the privilege does not encompass

14

everyone and everything in relation to it. This is a way to limit the


privilege as privilege is meant to help only the guilty people.)
Reasoni
ng

TEST
1.
2.
3.
4.
5.
6.

Reason
s

Comme
nts

Communications (versus facts)


Made by an employee
To Counsel representing the company
Acting as such (acting in the capacity of a lawyer for Upjohn)
Where the communications made at the direction of corporate superiors
In order to secure legal advice from Counsel
a) information not available to upper echelon employees
b) communications concerned matters within the scope of the employees
corporate duties
c) employees were aware that they were talking to lawyers so that the
company could obtain legal advice
d) request for information from lawyers was connected to advice giving to
the employees
e) the corporate echelon ordered that these communications be highly
confidential
f) the communications have in fact been kept confidential by the
corporation

Using control group method may not permit full disclosure of information
and thus not allow lawyer to provide appropriate legal advice frustrates
the purpose of the privilege
Information that came from non-management employees was required to
provide proper legal advice and obtain complete information
Work rule doctrine is not overcome memos from interviews are
privileged because they would be littered with notes from attorney
showing his mental process in collecting the information Fed R. Civ Pro
Rule 26 protection against revealing attorneys mental process

The 2 reasons for the attorney-client privilege:


1. Preserving observance of the law
When lawyers get all the information they need, they can give back
advice that people will actually follow
This is unlike the decision of Bentham that states it allows people to break
the law
2. It allows for some legal advice or advocacy
A lawyer can only argue for those objectives that a client has if the lawyer
is adequately informed. Thus, a lawyer can only give the court what it
needs if it gets all the information. Further, the lawyer will not waste the
courts time they will recognize where the law is and only argue
accordingly
Upjohn protects the downward flow of advice from lawyers and the upward flow
of information from those working at lower levels within the company
- The control group test may lead to a zone of silence within corporate
affairs which is why it was rejected in this ruling

15

Documents, Attorney-Client Privilege and the Bill of Rights


- Privilege extends to documents or portions of documents containing attorney-client
communications
BUT does attorney-client privilege protect against discovery of documents in the possession
of a lawyer, if the documents would have been protected by the Fifth Amendment, had they
remained in the possession of the client?
Fisher v. United States
- The owner of Fisher (co) had some financial books and a personal diary which he had
given to his lawyer
- The Justice Department issues subpoenas to the lawyer
- The lawyer responded by asserting the 5th Amendment (right against compelled selfincrimination)
- The lower court said the 5th Amendment flows through the client to the lawyer and
the lawyer can assert the 5th Amendment
- The US Supreme Court said this is wrong - Neither the lawyer nor the client would
enjoy the 5th Amendment in this case
a. The 5th Amendment does not protect the client because nothing is being
compelled from the client. It is being compelled from the lawyer
b. The 5th Amendment does not protect the lawyer because he cannot be
incriminated (the client would be)
- However, the attorney-client privilege would protect the material given to the lawyer
but ONLY the diary
- Although it was held that it does not violate the 5th Amendment, it DOES violate
attorney-client privilege and privilege may be used to protect the information only if
it is testimonial in nature
Corporate records and tax information were not testimonial in nature, and
were therefore held to not be protected
Examples: diary, letters, affidavits, etc.
- Written materials prepared in advance that were not prepared specifically for seeking
legal advice (pre-existing documents), are NOT covered by privilege
In addition, a tape recording between the client and some third person that is
given to the lawyer is NOT a communication between lawyer and client
- More difficult to compel required records with incriminating information but records
that are voluntarily created with self-incriminating information are compellable in
light of the 5th Amendment
- What is covered under the ruling of Fisher: Item/material whose possession
of which would communicate incriminating information about the client;
item created by the client to communicate to the lawyer and would be
incriminating if discovered. (Creates a small box for items that are not
specifically communication)
United States v. Hubbell
- essence is that prosecution issued subpoena asking for turn over of everything that
incriminates and the court said that is improper
- This is the testimonial aspect of these documents. There was no search warrant for
them
- Under Fisher, he need not produce these documents
- End Result: for information in documentation such as this, the prosecution is going to
have to obtain the information through searches rather than subpoenas. (Increases
the size of the box created for items in Fisher).
To obtain a search warrant, the government must have probable cause that a
crime has been committed and must describe with some particularity the
place to be searched

16

The Crime-Fraud Exception


- This exception DOES NOT apply to past crimes; only applies to ongoing and future
crimes [United States v. Zolin]
Fraud is an ongoing crime until rectified, which would generally entail
disclosure to the defrauded
Possession of stolen goods and being a fugitive from the law are ongoing
crimes
- Applies regardless of the fact that the clients lawyer is unaware of the clients intent
United States v. Chen
Facts
- Chens had business Sunrider. Would import and pay tariffs on items at
actual costs of the goods, but then had its comptroller, Jau, prepare
fictional invoices showing that the company paid much more for the
imported goods than it actually did.
Would revalue them for paying taxes as though they cost much more
than the value on which they paid the tariffs.
- An importer who pays higher import tariffs based on higher costs of the
goods will pay less income tax to IRS.
- Were able to take ~$90M from differences in reported values
- Then made disclosure to Customs indicating they had understated cost of
items they imported. Stated true cost (that which was stated in their tax
returns)
- Employee left Sunrider and provided info to Customs. Indicated that
lawyer had recommended making a disclosure to customs. No indication
that lawyer knew disclosure would be false.
- Government subpoenaed lawyers, and argued no privilege because of
crime-fraud exception.
- CLAIM: it was an ongoing crime and the lawyers knew about it
Issue
Is the information protected by attorney-client privilege?
Holding Information is not protected by attorney client privilege. Sunrider used attorneys
to make fraudulent statements and thus Crime-fraud exception applies
Ratio
Attorney-client privilege cannot be used to shield ongoing or intended future
criminal conduct
Reasoni Cannot use the actual document to show fraud but rather the Zolin procedure:
ng
- Test for invoking crime-fraud exception: reasonable cause to believe the
attorneys services were utilized in furtherance of the ongoing unlawful
scheme
- It is clients knowledge and intentions that are of paramount concern to
the application of the crime-fraud exception.it is irrelevant if the lawyers
were in the dark. 82
2 step process from United States v. Zolin [The Standard]
1. Government must satisfy the judge that there is a factual basis adequate
to support a good faith belief by a reasonable person that in camera
review of the materials may reveal evidence to establish the claim that
the crime-fraud exception applies
2. If the judge decides in favour of the government, the otherwise privileged
material may be submitted for in camera (in chambers / private)
examination

17

Notes
Review
Class

- To invoke the crime-fraud exception successfully, the government has the burden
of making a prima facie showing that the communications were in furtherance
of an intended or present illegality and that there is some relationship
between the communication and the illegality
- Whether there is reasonable cause to believe that the attorneys services were
utilized in furtherance of the ongoing unlawful scheme. Mere allegations or
suspicion by the government are insufficient. But Beyond a reasonable doubt
is not necessary to justify application of the crime-fraud exception.
- Rule 1.2(d) lawyer can advise about consequences but not assistance in doing
crime
o Have to make a motion, have to have that motion supported by something
other than the communication that your trying to get at to prove the crimefraud exception
o An example would be from the Clinton era, the Monica Lewinksy
scandal
-

In re Sealed Monica Lewinsky Case


o Lewinskys lawyers stated that they were protected under 5th
Amendment [Against Self Incrimination] did not have to submit
the affidavits to the government
o Lower court agreed Appeals Ct said no the affidavits were
privilege, made under fraud there was independent evidence for
a reasonable person to believe this. Therefore, no privilege
o Lawyer can raise attorney client privilege, based on had the
materials been in hand of the client they would be protected under
5th amendment
Class 2015: The government had multiplied its burden, all they had to show was
client had motive. They spent a lot of money on trying to prove lawyers
involvement. Government took a greater burden then needed.
- Doesnt have to be a violation of criminal law, it can be civil fraud, or
misrepresentation
- Proceeding = Privilege, No Proceeding = Confidentiality
Waiver of Attorney-Client Privilege
- A client loses the privilege with respect to a particular communication either by
consent or by conduct inconsistent with maintaining the privilege
o Only the client may waive the privilege; but, because lawyers have implied
authority to waive a clients confidentiality rights in the course of
representation, waiver may flow from a lawyers action even though the client
was not consulted
In re Columbia/HCA Healthcare Corp. Billing Practices Litigation
Dept. of Justice began investigating Columbia/HCA for possible fraud.
Facts
Columbia conducted several internal audits but when DOJ attempted to
obtain audits, Columbia claimed attorney-client privilege. But a change in
corporate control led the company to engage in negotiations with the
government, and produce some of this audit information but agreed that
this would not be considered as breaking attorney-client privilege.
Government agreed, In exchange for this cooperation, DOJ agreed that
certain stringent confidentiality provisions would govern its obtaining of the
documents. Contractual agreement

18

Issue
Holding

The various plaintiffs [private individuals & private insurance


companies]sued contend that like the Health Care Finance
Administration, Columbia/HCA overbilled them for various services
Does such a thing as a selective waiver exist?
Rejected the concept of selective waiver, in any of its various forms. The
client cannot be permitted to pick and choose among his opponents.
- Cannot use it as both a sword and a shield (it is either offensive or
defensive)

Ratio

Arguments

Comments

As a general rule, attorney-client privilege is waived by voluntary


disclosure of private communications by an individual or corporation
to third parties; can also be waived by conduct
- Once a client waives the privilege to one party, the privilege is
waived for all
Healthcare people say it has to be different when the government is
involved (the government should not be considered to be a stranger)
Healthcare people were simply being cooperative with the government
because they had been compelled to do so prior (this is why they
argued it should be different)
- They agree that privilege should apply to everybody, but there
should be special rules when you are dealing with the government
(because this would make it extremely easy for anyone with
authority in the corporation to be sued by shareholders)
No Selective Waivers: Promotes business being bad corporate citizens
because collecting any kind of information could eventually be suicide
(anything shareholders find out will go against the company and would
result in a lawsuit)
There is no general rule regarding wavier, but the dominant
view is, no selective waiver
Justification for the third-party waiver rule
o Designed to foster frank communication between attorney and
client
i. Absence of communication would leave the adversarial
process with no more information and with counsel less
able to present focused arguments to the court
- No Selective Waiver
- Client cannot be permitted to pick and choose among his
opponents, waiving the privilege as to some and resurrecting
the claim of confidentiality to obstruct others
- Selective Waiver in All Situations
- To hold otherwise may have the effect of thwarting the
developing procedure of corporations to employ independent
outside counsel to investigate and advise them in order to
protect stockholders, potential stockholders, and customers
- Some Selective Waiver
- Establishing a rigid rule would fail to anticipate situations in
which the disclosing party and the government may share a
common interest in developing legal theories and analyzing
information, or situations in which the SEC and the disclosing

19

party have entered into an explicit agreement that the SEC will
maintain the confidentiality of the disclosed material
Transactions with Persons Other than Clients
- The above case is about privilege. What about inadvertently released information?
i.e. a lawyer who receives a fax that was clearly not intended for him
Rule 4.4(b) A lawyer who receives a document or electronically stored information
relating to the representation of the lawyer's client and knows or reasonably should
know that the document or electronically stored information was inadvertently sent
shall promptly notify the sender
- Prior Rule: Lawyer should stop reading ASAP. Advise the sender they received it.
Destroy all matters or return to original sender
o This was changed waiver is waiver
o Even if the information was inadvertently waived, it has still been waived
o Now must communicate this information to the client. See rules 1.2, 1.3, 1.4
Model Rule 4.4(b)
- Where a document is sent that is privileged, you need to notify the sender to allow
them to take protective measures
- It is a matter of law put to the Court whether the privilege is waived and should
be picked up in a matter of litigation
- Whether they can notify the client is up to professional judgment see 1.2 and
1.4 (duty of communication)
ABA Model Rules
1.0 terminology
- What is defined
- 1.0(f) knowledge = actual knowledge
1.1 competence
1.2 scope of representation page 12
- if you are representing a client, you need to do certain things while representing that
client
- (a) setting up relationship between lawyer and client (lawyer is responsible for the
objectives, client is responsible for the goals)
- (c) may limit the scope of representationafter informed consent
1.3
1.4 duty of communication page 16
- Essence is that you have a duty to keep your client informed of what is going on in
the case and important developments in the case
1.5
1.6 Confidentiality of Information
1.8 (b) is related to confidentiality
1.7, 1.8, 1.9, 1.10 are all related to conflict of interest
1.9 (c) is also about confidentiality related to 1.6
- 1.9(c)(1): what can you say about someone you used to represent; the duty of
confidentiality is eternal (beyond death even) but the duty of loyalty is not
1.13 additional exceptions not included in 1.6

20

Organization as client (MAY report, NOT SHALL report option to report and withdraw
without criminal sanctions corporations as non-human clients)

1.18 prospective client rule: still covered if they are not happy with the advice and decide
not to become a client at that time, where there is reasonable expectation that they would
have become a client if you inadvertently blurt out information this is not protected
4.4(b) transactions with persons other than clients
8.4 (b) and (c) talk about how a lawyer should not engage in fraud or criminal activity that
reflects adversely on a lawyers fitness to practice
Other pages: 321, 323, 324 (Meyerhofer)

Professional Duty of Confidentiality


- The privilege allows a lawyer or client to refuse to give testimony on certain matters
in court or in another proceeding in which the evidence rules apply
- Protects 1) all info relating to representation whether learned before, during, or after
and 2) applied whether or not disclosure harm/embarrasses or not
- Duty of confidentiality has its roots in agency law
o An agent has a duty to the principal not to use or to communicate information
confidentially given him by the principal or acquired by him during the course
of or on account of his agency
Scope of Duty of Confidentiality
Model Rule 1.6 Confidentiality
a) Protects all information relating to the representation whether the lawyer learned
the information before, during or after the representation. It applies whether or not
disclosure would harm or embarrass the client. Also eliminates the word knowingly
from its prohibition, stating flatly that a lawyer SHALL NOT.
1.9(c)
- 1.9(c)(1) what can you say about somebody you used to represent (the duty of
confidentiality is eternal but the duty of loyalty is not)
- Duty of confidentiality extends beyond death
1.18 - duties to prospective clients
- Still covered if they are not happy with the advice and decide not to become a client
at that time where there is reasonable expectation that they would have become a
client
-

Controversial in listing the exceptions to the rule (in section B)


A lawyer is permitted to reveal confidential information when disclosure is impliedly
authorized to carry out the representation

Consequences of Breaching Confidentiality


- May result in professional discipline
- A lawyer can be sued for malpractice for violating a clients confidences providing
damages can be shown

21

Confidentiality and Joint Clients


- Any communications between one of the jointly represented clients and their counsel
remains privileged as against third parties (i.e. those who are strangers to the
attorney-joint clients relationship), but not as among the former joint clients
Model Rule 1.7 conflicts of interest
- Suggests that if one client in a joint representation insists that critical information be
kept from the other, the lawyer would be bound to keep that in confidence, albeit
while resigning from the joint representation
- (b)(4) informed consent and confirmed in writing (need definitions)
-

Can one client tell lawyer information and specifically hide it from joint client?
creates conflict
o Lawyer has duty to other client (must be loyal, diligent and communicate)
o BUT lawyer would compromise confidentiality with initial client and risk
impairing the level of trust therefore, at a minimum, must resign from the
joint representation
Lawyer should obtain advance waivers of confidentiality when
representing joint clients
In the absence of agreement among the joint clients and the
lawyer, there is a risk that the lawyer will be liable either to the
communicating client, for breaching the duty of confidentiality
(M.R. 1.0(e)), or to the other client for failure to communicate
material information (M.R. 1.4)

Case Law: A v. B
[GOOD HYPO]
o Law firm that jointly represented husband and wife when drafting a will,
learned that husband had an illegitimate child (information did not come from
child, but alternate source)
Firm wrote to the husband that it believed it had an ethical obligation
to inform he wife of the situation
o NJ S. Ct. held lawyer had option to disclose information about the child at
their discretion (note: here they were only dealing with whether the lawyer
could disclose).
Firm had discretion to make disclosure under New Jerseys version of
1.6 which permits, but does not require lawyers to reveal such
information (because in one sense, husband was committing fraud by
making a joint will, and not informing wife of potential split in assets
upon death)
o Another issue not addressed here: can the lawyer be held responsible as an
aider or abettor if he did not provide the information to the wife?
If you can be prosecuted as aiding and abetting does this change
your decision to inform the wife about the child?
IF A CONFLICT ARISES, SUCH THAT THE LAWYERS DUTIES OF
COMMUNICATION AND COMPETENCE REQUIRE COMMUNICATION
OF INFORMATION PROTECTED BY THE DUTY OF
CONFIDENTIALITY, THE LAWYER MUST WITHDRAW FROM
REPRESENTING BOTH CLIENS, AND MAY NOT COMMUNICATE
THE SUBSTANCE OF THE CONFIDENTIAL INFORMATION

Overview of Exceptions to Confidentiality


- Exceptions to the professional duty of confidentiality may be classified into three
broad areas:

22

o
o
o

Protection of lawyers threatened by a claim or charge brought by the client or


a third person (the self-defense exception)
Protection of innocent third parties who are being or may be victimized by the
client (defense of third parties)
Prevention or rectification of fraud on the tribunal

Self-Defense Exception
Model Rule 1.6(b)(5)
- Arises in three types of cases:
o When a client charges a lawyer with wrongdoing in the course of
representation
Client waives the attorney-client privilege by putting the lawyers
representation in issue
o When a lawyer sues the client to enforce some duty owed the lawyer, such as
payment of a fee, and
It is unfair for the beneficiary of a fiduciarys services to receive those
services and not perform duties owed to the fiduciary
o When a third person accuses a lawyer of wrongdoing in the course of
representing a client, perhaps in complicity with the client
Meyerhofer
- Meyerhofer v. Empire Fire and Marine Ins. Co.
Facts
Empire Fire made a public offering of 500,000 shares of stock at $16 a share on
March 28, 1972. Goldberg, working for the law firm realized that there was
misrepresentation of commission fees for brokering deals. Plaintiff later
purchased stock and lost money (along with other purchasers) and thus
brought a class action that prospectus was materially false and misleading.
Goldberg was later contacted to be a defendant in the case but he said that he
had worked for D and expressed concern over the excessive fees that were
not disclosed properly. Goldberg insisted on full and complete disclosure, but
the firm disagreed thus he resigned from the firm.
Later, he presented evidence to Plaintiff that resulting in Plaintiff adjusting
claims, adding more specific details, but the overall theory of the claim stayed
the same.
Originally brought a motion to disqualify!
Issue

- Was Goldberg justified in making this information public once he learned that he
had been included as a co-defendant?
Ratio
A lawyer may reveal confidences or secrets necessary to defend himself against
an accusation of wrongful conduct but disclosure has to be limited
Reasoni - Goldberg had the right to defend himself; the problem arises in the way in which
ng
he defended himself
- Goldberg overshared there was no need to give up information about a 3rd
party breaking confidentiality
Comme - Goldbergs disclosure violated Canons 4 and 9 of the Code of Professional
nts
Responsibility, but Plaintiffs law firm did not violate any of these
Rule 3.7 a lawyer who is witness before a jury cannot serve as a lawyer in that trial
1.0(f) actual knowledge is defined
Obtaining Confidential Information Improperly

23

Beiny v. Wynyard: prominent Wall Street firm gained privilege documents through
deception. Court ruled that the evidence should be suppressed and the firm
disqualified
- A lawyer encouraging another to breach client confidences is liable to be disqualified
from the relevant case and may face other serious sanctions
Client Fraud
- O.P.M.: A Case Study and its Lessons
- OPM (Other Peoples Money) was created in 1970 to purchase mainframe computers
and lease them to businesses
- Banks lended OPM the money to purchase these computers, and the loans were
secured by the leases. The leases would generate enough money to repay the loans
and leave OPM with a profit
- OPM became one of the largest computer leasing companies, but slashed its prices
and they stopped generating a profit. OPM retained the appearance of solvency
because of fraud
- Classic pyramid or ponzi scheme, where they lied to lender about the size of the
lease payments and about the existence of a lease that was to generate the money
to repay the loan
- Law firm of singer Hunter handled legal work, providing legal opinions to the lenders
concerning the soundness of the security to loans, knowing that OPM were capable of
serious illegality
- In June 1980, Clifton, a former CFO of OPM, had evidence that leases were fraudulent
- Goodman (major member of OPM) was at law office when letter from Clifton arrived,
and retrieved the letter before the lawyer could read it. Goodman left the firm and
told Hunter (another lawyer at Singer Hunter) that he was responsible for a wrong
- Hunter met with Cliftons lawyer, but did not request a copy of the letter or seek
details on the fraud
- Hunter and Singer obtained ethics consultancy, and told them they wanted to do
what was proper but also wanted to continue representing OPM
- Ethics people suggested 1) that the frauds were in the past prohibited Hunter and
Singer from reporting them, 2) that they had no direct knowledge of the frauds and
therefore did not have to stop representing them, and 3) Singer Hunter could
continue to close OPM lease dealings as long as Goodman provided a letter stating
the legitimacy of each new transaction
- Goodman told Singer hunter of the fraud in 1980, and the firm gradually withdrew
support and did not disclose OPMS ongoing frauds
- Singer Hutner dissolved as a result of the case and paid $10 million in malpractice
fees
o If all you know is the ethics rules, you do not know enough
o You have not factored in the illegal/fraudulent conduct and that
makes the lawyer an aider and abetter
o This must be figured out first before confidentiality/privilege
o CANNOT just focus on the ethics rules
Exceptions for Client-Fraud
-

A lawyer may reveal [confidential] information to the extent the lawyer reasonably
believes necessary:
o To prevent the client from committing a criminal or fraudulent act that the
lawyer reasonably believes is likely to result in . . . substantial injury to the
financial interests or property of another; [or]
o To rectify the consequences of a clients criminal or fraudulent act in the
furtherance of which the lawyers services had been used

24

Required Disclosure of Client Fraud under M.R. 4.1


Model Rule 4.1 Truthfulness in Statements to Others
- In the course of representing a client a lawyer shall not knowingly:
(a) Make a false statement of material fact or law to a third person; or
(b) Fail to disclose a material fact when disclosure is necessary to avoid assisting a
criminal act by a client, unless disclosure is prohibited by Rule 1.6
-

Revealing client fraud because a requirement in situations covered by Rule 4.1(b)

Confidentiality when Death or Bodily Harm May Result


Model Rule 1.6
- A lawyer may reveal information to the extent the lawyer reasonably believes
necessary:
a. To prevent reasonably certain death or substantial bodily harm
- Macumber
o A lawyer reported to public officials that his client, now deceased, had
confessed to a crime for which another person had been prosecuted and
convicted. The disclosure was ethically permissible
Hawkins: Rejecting a General Duty to Warn About Dangerous Clients
- Hawkins v. King County
o Hawkins arrested on minor drug charge and mother wanted him committed
because she believed he was mentally ill and a danger to himself and others
(mother and psychiatrist told this to Sanders)
o Sanders, Hawkins lawyer, following Hawkins advice and successfully
petitioned for Hawkins release on bail (did not provide information from
mother)
o 8 days after release, Hawkins assaulted his mother and attempted suicide
mother decided to file suit against Sanders
o Mother had 2 arguments
Sanders had a duty to tell the court about his clients mental state
He had a common law duty to warn foreseeable victims
o Court:
Rejected first claim
Regarding second claim: : Hawkins victims, his mother and sister,
knew he might be dangerous, thus no duty befell Sanders to warn
mother and sister of a risk they were already cognizant of
o Also, Sanders received no information that Hawkins planned to
assault anyone, only that he was dangerously ill and likely to be
dangerous
o Did not receive any information directly from Hawkins
Protecting the Judiciary from Dangerous Clients
- Attorneys, as officers of the court, have a duty to warn of true threats to harm a
judge made by a client or a third party when the attorney has a reasonable belief that
such threats are real (Washington v. Hansen, 862 P.2d 117, 122)
Protecting the Public from Dangerous Whistleblowers
- Washington Court disciplined a lawyer for disclosing client confidences that exposed
a corrupt judge
- Problem was that the lawyer disclosed client confidential communication to the press,
whereas he should have provided it to the appropriate authorities only

25

CHAPTER 5 CONFLICTS OF INTEREST


Purposes, Origin and Scope of Conflicts of Interest
- Loyalty requires putting the interests of another ahead of all other interests including
ones own interest and the interests of third parties outside the relationship, to whom
one might also owe loyalty. Whenever these interests materially diverge, a conflict of
interest exists
o Lawyers may face a conflict between their obligations to their client and their
obligations to the law
o The lawyers duty of confidentiality, itself an aspect of the duty of loyalty, is
grounded in the lawyers continual exposure to highly sensitive and valuable
information
o The duty of confidentiality is eternal; the duty of loyalty is not
Two adverse effects when there is a conflict of interest:
1. Impairment of the Lawyer-client relationship
o Erodes client trust
&/OR
2. There is a reduction in the quality of representation
o Short-changes the client
- In a serious conflict of interest it is likely that both will occur
4 underlying concerns of conflicts of interest:
1. The trust aspect of loyalty (1.7(a))
o The lawyers rep deteriorates because of antagonism, mistrust, etc. by client
2. The commitment aspect of loyalty (1.7(b))
o Less zeal, diminished diligence, less effectiveness
3. The breach of duty of confidentiality (1.6)
o Confidential info being used for a former client
4. The appearance of impropriety
o This was a standard in the old code but it was abandoned when the rules were
adopted
o More of a historical concern (but still here today)
2 Broad categories of conflicts of interest
1. Conflicts between the lawyer and the client
2. Conflicts between clients themselves
a. Concurrent representation conflicts (conflicts between 2 or more lawyers
current clients. Rule 1.6)
Need consent for concurrent conflicts
However, need to be able to get this consent under the rules
There are some non-waivable/unconsentable conflicts that exist
b. Successive representation conflicts (between a current client and a former
client. Rule 1.9)
A consent cures circumstance
There are no non-waivable, non-consentable former client conflicts that
exist
If you get consent, any ethical issue is cured
1.9 was intended to not include or give benefits to former clients bc of
loyalty; case law says otherwise

26

THE TWO ARE TREATED DIFFERENTLY BECAUSE:


Duty of confidentiality is eternal; the duty of loyalty is not
Concurrent clients owed BOTH duty of loyalty and duty of
confidence
Successive clients are ONLY owed a duty of confidence and not
loyalty
o Therefore, can sue a former client (just not a present
one)

Eternalness of Confidentiality
- Swidler
o Foster killed himself
o Firm (Swidler v Berlin) went to his lawyers and wanted all the info Foster had
told them
o Claim: When the client is dead, confidentiality ceases when it comes to
national security
o Court: No the duty of confidentiality is eternal regardless of the
situation
Model Rule 1.16 Withdrawal of a lawyer (DISCUSSED IN CLASS)
(b) May withdraw for other circumstances
- One is a client failing to live up to its obligations
OR may withdraw if it is financially burdensome
Statute of limitations in most jurisdictions will govern how long that you have to sue but
there are reasons why you would not want to withdraw
- Malpractice statute of limitations is 2 years in Michigan can still countersue for
malpractice if past the 2 year limitation, but damages are limited
- Contract statute of limitations is 6 years in Michigan
Professional Rules Governing Conflicts of Interest
Model Rule 1.7 Concurrent Conflicts
- You need consent + something else (covered by (b))
- 1.7(b) is informed consent in writing
Model Rule 1.8 Conflicts between a clients interests and the lawyers interests
Model Rule 1.9 Successive Conflicts
- Any consent from a client cures
Model Rule 1.10 Imputation
Other rules address conflicts problems for specific types of lawyers, such as former
government lawyers (M.R. 1.11), former judges or arbitrators (M.R. 1.12), problems for
specific types of clients, such as organizations (M.R. 1.13(f), (g)) or prospective clients (M.R.
1.18), or conflicts arising in particular situations, such as opinion letters (M.R. 2.3), and
lawyers acting as transactional intermediaries (M.R. 2.2 eliminated)
Concurrent Representation in Litigation
Professional Rules on Concurrent Representation
- Model Rule 1.7 identifies two different types of concurrent conflicts in litigation, either
of which disqualifies the lawyer from representing both clients

27

o Representations that are directly averse (Rule 1.7(a)(1))


Lawyer is prohibited from representing a client if the representation will directly averse
another client
- The paradigm of direct adversity is one client suing another: a lawyer may not,
without consent, represent her clients opponent in any matter, no matter how
unrelated to the clients suit
- A lawyer cannot cross-examine a client who appears as a witness in a lawsuit
involving another client
o Direct adversity focuses on the likely incentives and beliefs of the client, and
the risk that the client will, as a result of the lawyers conflict of interest, act in
ways detrimental to the clients interests
- Representations that are materially limited (Rule 1.7(a)(2))
o The focus of the material limitation branch is on diverging interests that are
likely to have a harmful impact on the quality of the representation of one or
more clients (focus is on lawyers incentives to act, as a result of a conflict of
interest, in ways potentially detrimental to one or more clients interests)
Consent and Concurrent Representation Conflicts
- Three categories of nonconsentable conflicts:
o Conflicts are nonconsentable if the representation is prohibited by law
MR 1.7 (b)(2); Restatement 122(2)(a)
o Conflicts are nonconsentable if the lawyer is representing one client who
asserts a claim against another client of the lawyer in the same litigation
MR 1.7(b)(3); Restatement 122(2)(b)
o The conflict is nonconsentable if the risk that the representation will be
affected by the conflict is too great
MR 1.7(b)(1); Restatement 122(2)(c)
- What constitutes sufficient consent?
o Each affected client must give informed consent
WHAT IS INFORMED CONSENT: Informed consent means that the
lawyer has communicated adequate information and explanation about
the material risks of and reasonably available alternatives to the
concurrent representation
Requires consent to be confirmed in writing (1.7(b)(4))
includes electronic communications such as e-mail
Concurrent Representation in Civil Litigation
Westinghouse Elec. Corp. v. Kerr-McGee Corp.
Facts
Major law firm Kirkland and Ellis worked with both the American Petroleum
Institute (a consortium that included the Appellants, Kerr-McGee Corp.) and
Westinghouse on two separate matters.
These two interests were put in serious conflict when the firm released a report
on behalf of the API finding trade practices to be generally acceptable in the
uranium industry on the same day that it filed suit on behalf of Westinghouse
alleging an ongoing illegal conspiracy in restraint of trade in the same
industry.
In the course of preparing this report, Kirklands lawyers had sent out a
questionnaire to employees of API member companies, including the
Appellants, with the understanding that the results would be held in
confidence.
Issue
(1) Whether an attorney-client relationship arises only when both parties
consent to its formation or can it also occur when the lay party submits
confidential information to the law party with reasonable belief that the

28

latter is acting as the formers attorney and


- Westinghouse is being represented by Kirkland
(2) Whether the size and geographical scope of a law firm exempt it from the
ordinary ethical considerations applicable to lawyers generally
- API lobby against diversification of energy/uranium industry
- Is Kirkland disqualified because there is a conflict of interest in
handling the two matters (because it represents both Westinghouse
and API)
(1) Dont necessarily need express consent to form an attorneyclient relationship
(2) Size and scope should not exempt it from ordinary ethical
considerations
- District court said: there existed no explicit or express attorney-client
relationship in that no oil company representative requested Kirkland to
act as its attorney orally or in writing and Kirkland did not accept such
employment orally or in writing (basically said, if you are not literally a
client, then you are not protected formalistic approach)
- Circuit court took a more functional approach: certain
expectation of confidentiality in the dealings
- A professional relationship is NOT dependent on the payment of
fees nor upon the execution of a formal contract
- Agency test:
- The power to affect the legal relations of the principal and others
- A fiduciary who works on behalf of his principal and primarily for his
benefit
- And a principal who has the right to control the conduct of the
agent
- None of these existed
- Court took into account the fact that it was a large law firm (difficult not
to have conflicts of interest)
- The professional relationship for purposes of the privilege for attorneyclient communications hinges upon the clients belief that he is consulting
a lawyer in that capacity and his manifested intention to seek
professional legal advice
- Is there anything Kirkland could have done to save it from disqualification
bring in a third party consultant to obtain the industry information from
API so that they dont think that Kirkland is looking to represent them
(this courts main interest is the CONFIDENTIALITY CONCERNS)

Holding

Reasoni
ng

Comme
nts

Merely hatred between two parties is sufficient under ABA


1) direct adversity or 2) material relation in 1.7(b) is required

- who is the client not defined in the ABA and covers broader spectrum than specific person
paying bills
Rule 1.7 Comment 6:
Loyalty to a current client prohibits undertaking representation directly adverse
to that client without that client's informed consent. Thus, absent consent, a
lawyer may not act as an advocate in one matter against a person the lawyer
represents in some other matter, even when the matters are wholly unrelated.

29

Exampl
e

Issue: how long does loyalty to a client last?


Unlike confidentiality, it is not eternal
Situation 1:
- P v. D, where lawyer represents P
Situation 2:
- Bank v. P (foreclosure), where lawyer represents Bank
The loyalty in Situation 1 for the client (P) would disqualify lawyer from
being able to represent Bank from Situation 2. Even if consent from the
client is given, there is a conflict of interest

Fiandaca v. Cunningham
Facts
NHLA represented female prisoners looking for equity in prisons (wanted same
privileges as men). NHLA also represented mentally challenged people in
Garrity. Prisoners were looking for new facilities to be built and got offered a
place at Laconia State School (where the people in the Garrity litigation reside.
NHLA did not want to make a deal that was against the interests of Garrity so
they rejected the offer, even though it was in the interest of the female
prisoners. The state countered by wanting to disqualify NHLA as class counsel
because of unresolveable conflict, but this was denied due to the case already
being going on for 3 years. Court decided to go through with the LSS location so
NHLA withdraws. The settlement agreement the court wanted didnt end up
going through so NHLA wanted to be reinstated as counsel but court denied this
motion on the grounds that necessity of participation by NHLA no longer
existed because issue had been tried to conclusion. Female prisoners are now
appealing NHLAs ability to represent them after the settlement fell through.
Issue
Should NHLA be disqualified from this litigation due to the inherent
conflict of interest that arises from representing these two parties?
Holding
Yes. The district court mistakenly ignored the serious conflict of interests
presented in this case. Although this conflict probably didnt seriously affect the
trial proceedings, it undoubtedly affected NHLAs actions in the remedial stage.
As such, the district courts remedial order is vacated and the case is remanded
for a new trial to determine the proper remedy for the prisoners Constitutional
deprivation.
Reasoni
- NHLA had an ethical duty to prevent its loyalties to other client
ng
from coloring its representation of the plaintiffs in this action
and from infringing upon the exercise of its professional
judgment and responsibilities (could have solved the whole
matter if LSS wasnt NHLAs client as well)
- NHLA also has to fairly represent LSS who do not want prisoners on their
facility
Theory:
- If you are a lawyer in a position to violate the rules of professional
conduct, you are violating public policy
- If you are violating public policy, anybody can stand up and say this is
wrong
- Argument: Law firm has a non-waivable conflict of interest
- Under the current 1.7 analysis there at least a material limitation
meaning they cannot serve one side as zealously than the other (1.7(2)).
There may even be direct adversity.

30

NHs theory: The consent is not possible because of the


unreasonableness of the representation
Can they really consent to anything? the Court somewhat ignores this

Curing a Simultaneous Conflict: The Hot Potato Doctrine vs. Thrust-Upon Conflict
- Picker International, Inc. c. Varian Associates, Inc.
o Jones Day firm merged with smaller firm and one of Jones Days long-time
client was about to sue one of its new clients which the smaller firm had
previously been representing on unrelated matters
o Concurrent conflict could not be cured by withdrawing from representing the
new client, especially if it is in order to keep the more lucrative client (should
have withdrawn from the client prior to the merger)
Hot Potato Doctrine
- An existing client and a new client comes in and they are going to be adverse to each
other, and you drop the first client to retain the second client
- Dropping the less-desirable Client
- Changes from a concurrent conflict to a successive conflict
- The issue is that you do not want to allow a law firm to create a successive
representation consideration of a conflicts issue when the real issue should have
been concurrent representation
[m1] A v B A is your client
[m2] C v D C is your client
[m3] comes along C v A in order to take on A, you drop C
o C can say even if there is no confidential info from any case which is
at all relevant to the others, because of the direct adversity, C can
say you cannot represent A against me
- If a law firm has two clients and the conflict is there and it fires one of those clients,
then when client C makes its disqualification motion the law firm will argue that the
firm owes it only confidentiality and not loyalty; the client will get the benefit of the
doubt related to loyalty
o Use a 1.7 analysis
Idea that you have a situation where: you are trying to drop an
existing client in order to make that person or entity into a
former client for conflicts purposes (trying to get a different
test which would apply to you, so as to not violate any rules)
o prohibited from even taking the second party as a client
under 1.7(a)
Thrust-Upon Doctrine
- Competing doctrine to the hot potato doctrine
- Somebody has set you up to have a conflict; therefore, you can avoid it
[m1] A v B - you rep A
[m2] C v D you rep C
- C buys B
- But for Cs purchase of B, you can drop C
- Had they dropped A and represented C (the new B) this would be hot potato
-

The analysis can be a 1.9 analysis because you cannot have the loyalty issues
created from the outside and the loyalty issues thrust upon you like that
o Protection from 1.7

Remedies for Conflicts of Interest Violations


- Can come in two forms:

31

Against the Lawyer


Forfeiture of the fee to the client because of a breach of duty owed to
the client
Disqualification as a lawyer in that matter
Possible malpractice liability to the client for breach of standard of care
Criminal liability in serious circumstances
Against the Client
Rescission of a transaction
Dismissal/reopening of a claim or proceeding
Cost of new counsel, especially in a situation where work product made
by disqualified firm is not permitted to be used by new counsel
Look at the nature of the legal work
Look at whether the work may have been based on confidential
info of former client
Look at obviousness of conflict and reasonableness of
disqualification
Timeliness of the motion for disqualification

Concurrent Representation in Criminal Litigation


Cuyler v. Sullivan
Facts
Respondent, John Sullivan, was convicted of a double murder relating to a union
dispute and sentenced to life imprisonment. He had been indicted with two other
defendants, and all three were represented by two privately retained attorneys.
Sullivans trial, however, occurred before the other two defendants, and there
was evidence on the record to suggest that his attorneys may have purposely
rested their case earlier than necessary in order to avoid exposing too much of
their strategy in advance of the two remaining trials. Sullivan is now seeking a
federal writ of habeas corpus, claiming that his retained defense counsel
represented potentially conflicting interests.
Issue
Whether a state prisoner may obtain a federal writ of habeas corpus by
showing that his retained defense counsel represented potentially
conflicting interests
Holding No, a state trial judge has no such obligation. Absent a request to do so from a
party, a trial judge is not required to question the propriety of multiple
defendants being represented by the same counsel.
Ratio
Defendants alleging a conflict of interest in cases in which multiple
defendants are being represented by the same attorney in the same
matter MUST DEMONSTRATE AN ACTUAL CONFLICT THAT ADVERSELY
AFFECTED THE OUTCOME.
Reasoni
- Unless the trial court knows or reasonably should know that a particular
ng
conflict exists, the court need not initiate an inquiry (not the case here)
Comme
- 6th amendment: in all criminal prosecutions, the accused shall enjoy the
nts
right . . . to have the Assistance of Counsel for his defense. (This is the
right to EFFECTIVE counsel)
- THREE FACTOR TEST to address whether to make an inquiry as to
whether or not there is a conflict of interest in criminal matters
for the basis of disqualification under the 6 th Amendment:
1. Are Ds being tried together or separately?
2. Whether or not the line of questioning in the defense is

32

consistent among the jointly representing Ds?


3. Whether the line of defense is based on reasonable and
tactical decisions and not motivated by conflict
These factors are not without criticism
o The trial judge cannot know the answer to #3 really until
the case is over
o Also true with #2 if the trial gets to this point, it is too
late
Thus, this holding has been subject to critism
The issues of joint representation in criminal matter are handled as
follows:
- If not raised by defendants, the trail judge need not make an
inquiry (Cuyler)
- If question is raised, the judge must make inquiry and may take
appropriate action (Holloway)
- If the judge does inquire, the judge has broad discretion to order
separate representation (Wheat)
- Disqualification order is not immediately appealable, and can only
be appealed on appeal of conviction (Flannagan)
- Defendant must show actual conflict of interest (Cuyler)

- The attorneys felt that they didnt need to; they thought they had the winning
case
- The states are not covered by FRE 44(c), case law does. Holloway Cant
ignore the issue, the court must inquire
Issues of Joint Representation in Criminal Matters: RULEs from Cuyler, Holloway,
Wheat, Flannighan, etc.
1. If the jointly represented Ds or their lawyers do not raise the issue of
conflict of interest, the trial judge need not make an inquiry (respecting
client consent) (Cuyler)
2. If the question is raised, the judge must inquire and may take appropriate
action (Holloway)
3. If the judge does inquire, either on her own motion or bc the matter is
raised by a party, the judge had broad discretion to order separate
representation (Wheat)
4. A disqualification order is not immediately appealable but may be
considered only with an appeal of conviction (Flannigan)
5. To upset the conviction on a collateral attack, the convicted D must show
an actual conflict of interest that affected the decision (Cuyler)
Mickens v. Taylor
o What happens if the trial court knows or reasonably should know that a
particular conflict exists, but the trial court fails to initiate an inquiry into the
conflict?
o Counsel for a defendant convicted of capital murder had represented the
murder victim for ten days preceding his death in assault charges brought
against the victim by his mother is this a conflict of interest?

33

Defence counsel did not believe he had any continuing duties to former
client, and so did not inform the defendant of the prior representation
Defense counsel instead represented the client in a manner that
showed he was still partially loyal to former client (did not pursue the
argument that the occurrence was consensual; did not put forward
character evidence about the victim)
Lightened the standard to show (prejudice is presumed), when it is concurrent,
it will be applied

Joint Representation in Transactions and Civil Litigation


- Lower the transaction costs to clients involved
- Lead clients to focus on their shared goals and interests rather than differences and
disagreements
Model Rule 1.7 Comments 8, 26 and 33, specifically
Representing Both Parties to a Real Estate Transaction
State v. Callahan Discipline Case
Facts
Respondent, attorney John Callahan, was acting as personal attorney to Lowell
Lygrisse. Lygrisse was looking to buy land owned by California resident Ruth
Fulton, and suggested to her that Respondent handle the transaction for both of
them. She agreed, and Respondent successfully drew the appropriate papers
(including a very unusual arrangement that included an unsecured, interest-free
loan as part of the purchase price that Fulton did not fully understand) and
closed the sale as per prearranged terms. When Lygrisse defaulted on the final
payment, Fulton came to the Respondent for advice. Respondent repeatedly
advised Fulton that Lygrisse would pay. When he did not, she retained separate
counsel for a malpractice action against Respondent once it became clear that
the sale agreement had given them no security interest in the property (it was
only a promissory note). Respondent argues that he was merely acting as a
scrivener for the two parties, and that he had no obligation to suggest better
terms than those provided by Lygrisse.
Issue
Was Respondents conduct proper?
Did Callahan have a duty to disclose the previous relationship with Legrisse?
Holding A lawyer seeking to represent multiple parties in a transaction with
adverse interests must obtain the consent of each other after full
disclosure. A lawyer cannot engage in dishonest, fraudulent behaviour.
Callahans conduct rose to the level of deceit when he did not advise
Fulton about her security interest in the property.
Ratio
An attorney must fully disclose all potential conflicts of interest to both
parties.
Reasoni CALLAHAN had a duty to
ng
- Disclose the previous relationship
o Would Legrisse have to give permission to disclose this info?
o Look up informed consent and terms in writing in rules
o Definitional section (1.0) of the Rules themselves
- Tell Fulton that there are risks associated with a joint
representation in all joint rep circumstances there are always
risks and this should always be brought to the attention of the
clients
- Special risk in this particular transaction there is no security on the

34

transaction (this is what the BAR in Kansas goes after Callahan for)

Comme
nts

WHAT IF
- All of these terms were explained to Mrs. Fulton and she was a
sophisticated businessperson?
- They told her you are only getting a promissory note, not a
mortgage?
- There is rick with all joint representation
- Assuming all risks and issues were disclosed could Fulton still go forward?
o She could but it should also be disclosed that this is a bad deal.
This is a responsibility of Callahan to make sure she knows this is a
bad deal. A reasonable lawyer would have indicated this to their
client and likely would have tried to prevent their client from going
forward.
- Court says that as long as Fulton suffers as a result of this deal, Callahan
is responsible for that.
- Considering Callahans knowledge of Lygrisses financial state and the
confidentiality that binds Callahans representation, Callahan cannot fully
represent Mrs. Fulton in this matter because he cannot provide her with
the best service possible. (without Lygrisses consent, he cannot disclose
the information about Lygrisses past financial state with Fulton due to
confidentiality, that is why he cannot fairly represent Fulton)
- (1) didnt disclose financial partnership
- (2) Didnt disclose the risks associated with joint representation
- (3) Didnt disclose and describe the special risk of the terms being so onesided
- Any time you have a 1.7 problem, chances are really good that
you have not designed means to the representation that are
appropriate to the goals your client had

Representing Buyer and Seller in a Real Estate Transaction


- Lawyer may represent buyer and seller in only some cases (like routine residential
real estate transactions) if certain conditions are met
- In Michigan and under the ABA rules it is permitted
- However, must meet the conditions under 1.7
o At the minimum, in order to represent them you must tell them that
joint representation is risky there are things that would come up had
there not been joint rep
o Because of the risk of conflict of interest (it is likely that there are material
limitations or direct adversity under 1.7(a)(1))
Lawyer Serving as Third Party Neutral
Model Rule 2.4 addresses the lawyers responsibilities when a lawyer acts as an arbitrator,
mediator, or other third-party neutral to assist parties to resolve a dispute outside of
litigation (only applies when the disputing parties are not clients of the lawyer)
Joint Representation in Marital Matters
- Joint representation invites challenges to marital agreements on the ground of
overreaching
- A single lawyer cannot represent both spouses in the courtroom phase of divorce
cases

35

Collaborative lawyering each party is separately represented, but the lawyers and
the parties all agree to resolving the dispute by negotiation, without filing a lawsuit
o The parties agree that if litigation is necessary, the lawyers will withdraw from
the representation and the parties will obtain substitute counsel
- To avoid disputes when arranging a pre-nup, it is better to bring in another lawyer to
represent the other party
Estate Planning
- Joint representation is common with the preparation of wills, trust instruments, and
other estate planning documents for a husband and wife who want to plan jointly for
the distribution of their assets after their death
Joint Representation in the Formation of a Business
- Individuals each pay a part of the lawyers fee and establish a personal lawyer-client
relationship on the common matter
Aggregate Settlement Rule
Model Rule 1.8(g) provides that a lawyer participating in the aggregate settlement of clients
claims must obtain the informed consent of each client in writing signed by the client
- Disclosure shall include the existence and nature of all the claims or please involved
and of the participation of each person in the settlement
NOTE: Any time you are taking confirmation from the client (especially in reference to
consent for conflict of interest), IT MUST BE CONFIRMED IN WRITING!
Also: keep in mind that the reasonable lawyer test will always occur in hindsight
Successive Representation
- General rule (found in Model Rule 1.9(a)):
o A lawyer who has formerly represented a client in a matter shall not represent
another person in the same or a substantially related matter in which that
persons interests are materially adverse to the interests of the former client
unless the former client gives informed consent, confirmed in writing
Need consent because the concern is that the lawyer may be tempted
to reveal or use confidential information from the former
representation in the current representation
TEST:
o Is there someone who qualifies as a former client?
o Are the interest materially adverse?
o Are the matters substantially related?
o Is there consent after consultation?
o Where there is disqualification, is that disqualification
imputed to the rest of the firm?
Model Rule 1.9(c) is a reminder that the lawyer has a continuing duty of confidentiality to a
former client even with consent provided under 1.9(a) this duty of confidentiality continues
Model Rule 1.11(a) lawyers whose former client is the government are governed by this
rule
Successive Representation of Joint Clients
Brennans Inc. v. Brennans Restaurants, Inc.
Facts
- Attorney Edward Wegmann was retained as general counsel for the Brennan
familys New Orleans restaurant chain and assisted them in obtaining a
federally registered trademark.
- When a management dispute forced the corporation to split into two
separate businesses, Wegmann was kept on by the Defendants to defend

36

their right to use the trademark against the Plaintiffs challenges.


Wegmann retained the services of Arnold Sprung, a New York trademark
attorney, to assist him in the case, and the Plaintiffs challenged Wegmanns
(and, by imputation, Sprungs) participation in the case on the grounds that
his representation of Defendants was at odds with his former representation
of Plaintiffs.
- The District Court agreed, and granted the motion to disqualify, which
Defendants now challenge.
- Should attorney Wegmann have been disqualified from this suit due to his
prior joint representation of Plaintiffs and Defendants?
- Should attorney Sprung have been disqualified from this suit due to his
association with Mr. Wegmann?
- The lawyers ethical duty of confidentiality is stronger and broader than the
attorney-client privilege. Yes, attorney Wegmann was properly disqualified
from this suit. As a former attorney for joint clients who are now opposing
parties, he will not be able to properly fulfill his ethical obligation of
confidentiality.
- No, attorney Sprung should not have been disqualified. He was never
counsel for Plaintiffs, and it was plain error for the court to disqualify him by
imputation, as the appearance of impropriety was not enough here.
- This holding is one of the strongest applications of the lawyers ethical duty
of confidentiality.
IN THE END, RECOGNIZE THAT THIS CASE IS NOT ABOUT
CONFIDENTIALITY, BUT IT IS ABOUT LOYALTY (a client would feel
wronged if an opponent won against him, if counsel was part of the
initial representation)
- This was not about confidentiality since S was allowed to stay
- One company: what secrets? There is no confidentiality
Footnote 7 p. 467 Wegman will likely be a witness Rule 3.7
5 issues to look for in successive representation/Conflict of interest crietia:
1. Is there a former client, person or lawyer that was formerly represented;
if not, was there someone who was in that former capacity?
2. Are there materially adverse interests? (is the former clients interests
materially adverse to the current buyer?)
3. What is the relationship of the two matters? (Are they the same or
substantially related?)
4. Is there consent from the former client?
5. Is the disqualification personal or is it imputed to the other lawyers in the
firm?
-

Issue

Holding

Reasoni
ng

Commen
ts

Model Rule 3.7 GOOD FOR EXAM HYPOTHETICALS!!


- A lawyer can be involved in a proceeding up until the lawyer is going to have to
testify (up until trial)
- General rule: A lawyer is not disqualified merely because the lawyer is going to be a
witness
- There is no question in Brenn
- ans that W was going to be a witness at some point, but under the ABA rules he did
not have to be disqualified until trial
- Look at comment 29 of Model Rule 1.7!!
Sample Exam Hypothetical:
- L is retained by A to do opening work for restaurant

37

L incorporates restaurant, negotiates and reviews terms of the lease, works out
credit arrangements with the bank and helps A get a registered trade name for the
restaurant
Six months after Ls work is done and L has been paid for her services, L is consulted
by B. B wants to start a competing restaurant across the street from As restaurant.
o May L represent B?
If it is only starting a new restaurant, then L can do this
A and B are merely economically adverse so there is no conflict.
Former client may feel wronged, but there is no rule of
professional conduct.
If A consulted you as to where to place the restaurant, and then B
came to you, this would be different, and a conflict of interest would
exist
A meat supplier, C, comes to L and wants to bring an action against A for not paying
the bills. As defense in the matter is that some of the meat was defective.
o May L represent C?
A is a former client. There are materially adverse interests. The
relationship between the two matters is NOT the same or substantially
the same.
You can sue the former client, but there are some matters you are not
allowed to take on this is probably one you CAN take on
i.e. say it was an eviction for breach of the covenant against
excessive noise and C was the landlord L would not be able to
represent C in evicting A because the relationships between the
two matters IS the same or substantially the same (because L
negotiated that lease with the landlord)

L- AB [Car Accident] Cant represent A or B together if they have cause of action against
each other. Rule 1.7(a) & 1.7 (b)
Ballet Dance wants to be represented by L for contract negotiations. At same time, New York
City Ballet, who the contact negotiations are with, wants L to represent in real estate deal
- You know that New York City Ballet is moving, can use it to negotiate with contract
- Debatable. There is no direct conflict
- If conflict, case law states that he would have to drop both clients
Other Successive Representation Issues
Substantially Related Matters
- Re American Airlines
o To be substantially related, the prior representation need only be akin to the
present action in a way reasonable persons would understand as important to
the issues involved
- Analytica, Inc. v. NPD Research, Inc.
o A matter is substantially related if the lawyer could have obtained confidential
information in the first representation that would be relevant to the second
- Government of India v. Cook Industries, Inc.
o To count as substantially related, the relationship between the two matters
must be patently clear, the issues identical or essentially the same
Model Rule 1.9 [comment 3]
- Matters are substantially related if they involve the same transaction or legal dispute
or if there otherwise is a substantial risk that confidential factual information as
would normally have been contained in the prior representation would materially
advance the clients position in the subsequent matter

38

A lawyer who recurrently handled a type of problem for a former client is not
precluded from later representing another client in a factually distinct problem of that
type
- In the case of an organizational client, general knowledge of the clients policies and
practices ordinarily will not preclude a subsequent representation
Competition as an Adverse Interest
- Model Rule 1.7 [comment 6]
o Simultaneous representation in unrelated matters of clients whose interests
are only economically adverse, such as representation of competing economic
enterprises in unrelated litigation, does not ordinarily constitute a conflict of
interest and thus may not require consent of the respective clients
- Whether a law firm can later represent competitors of its former client is a matter
that must be decided from case to case and depends on a number of factors.
o One factor is the extent to which the fiduciary was involved in its clients
affairs
Imputation and Lawyers Not in the Same Firm
Model Rule 1.10 only applies to lawyers in firms
- Must present themselves to the public in a way that suggests that they are a firm or
conduct themselves as a firm
Taint Shopping and Duties to Prospective Clients
- Taint shopping refers to the problem of a potential client contacting a lawyer and
revealing confidential information to the lawyer, but then not hiring the lawyer, solely
for the purpose of ensuring that the lawyer will not be available to represent the
potential clients adversary
o This happened in Westinghouse, but the court held that the defendant had
never been a client of the firm, only of the lawyer consulted therefore no
passing of information to his partners
Model Rule 1.18 addresses the problem of duties to prospective clients
- If a person discusses with a lawyer the possibility of representation, that person is a
prospective client entitled to the protections of the rule
- The lawyer who consults with the prospective client is disqualified if the substantial
relationship test is met and also If the information received by the lawyer could be
significantly harmful to the prospective client (higher burden on client to show this)
Imputed Disqualification and the Migratory Lawyer
Rules and Problems
Model Rule 1.10(a) if any lawyer in a firm is disqualified from representing a client or being
involved in a case, all the lawyers in the firm are disqualified
- Rationale for this rule:
o Lawyers who practice together talk to one another about their cases, and in
doing so share client confidences
o Lawyers who practice together share professional and financial interests,
which may tempt them to share information about one client for the benefit of
another
o Regardless of whether lawyers actually share particular information
improperly, that prospect could cause clients and the public to lose confidence
in lawyers generally
Model Rule 1.9 [comment 5] if a lawyer while with one firm acquired no knowledge or
information relating to a particular client of the firm, and that lawyer later joined another
firm, neither the lawyer individually nor the second firm is disqualified from representing
another client in the same or a related matter even though the interests of the two clients

39

conflict.

Nemours Foundation v. Gilbane


Facts
- Plaintiff requests disqualification of Defendant Gilbanes law firm, Biggs
and Battaglia [Ba-tah-ge-la], due to attorney Paul Bradleys participation
in a previous action in which he represented a co-defendant of Plaintiffs
- This request was made near the end of a very long and complicated case
involving a dispute over the construction of a new addition to a hospital
Issue
- Should attorney Bradley be disqualified from participation in this case?
- If so, should the entire law firm be disqualified by imputation?
Holding
Bradley is disqualified, but Biggs is not
- Attorney Bradley is disqualified due to his previous work on behalf of a
co-defendant of Plaintiffs. His continued representation of Defendant
would be materially adverse to his former client. He had a fiduciary
duty or implied duty because he had access to some confidential
documents when working on the Furlow case at Berg
- Biggs: While an attorneys disqualification is usually imputed to the other
attorneys in his firm, the court found that the Rules new exception to this
standard applies here. As long as the disqualified lawyer is
screened from any participation in the matter and receives no
share of the fee from it, the firm may continue its
representation.
- It is really about loyalty issue
Ratio

Reasoni
ng

Attorneys may be disqualified from representation of a client if such


representation would be materially adverse to a former client, but the firm
with which the attorney is associated may avoid disqualification if it ensures that
the disqualified attorney
- We know this is a loyalty case as opposed to a confidentiality
case because Biggs firm is allowed to stay (and Bradley has
been disqualified)
Issue #1
- Nemours argues that conflict of interest originates with Bradley
- Pierce argues Model Rules 1.6 and 1.9 Nemours must prove that
confidential information actually DID pass from Nemours to Bradley
- Nemours must be considered a former client of Bradley for the
purpose of determining whether a conflict of interest exists
- Because Bradley viewed documentation, whether he remembered it
or not, an attorney-client relationship existed
- Bradley should clearly be disqualified
Issue #2
- Pierce argues that an effective screening mechanism is an acceptable
alternative to disqualification of an entire law firm Model Rule 1.11
- Nemours does not present any disadvantage or harm that could come
with Biggs representing Pierce
- Court weighs factors and decides that Biggs should NOT be
disqualified

Commen

Model Rule 1.9

40

ts

The lawyer must have had an attorney-client relationship with the former
client
The present clients matter must either be the same as the matter the
lawyer worked on for the first client, or a substantially related matter
The interests of the second client must be materially adverse to the
interests of the former client
The former client must not have consented to the representation after
consultation

Model Rule 1.7


- Provides guidance for determining when the interests of two clients are
adverse
Model Rule 1.11
- A firm with which a former government officer/employee is associated
may undertake or continue representation in the matter only if the
disqualified lawyer is screened from any participation in the matter and is
apportioned no part of the fee therefrom
Model Rule 1.10(a)(1)
- The prohibition of the firm where one lawyer in the firm has been
disqualified, the exceptions are: Screening

Conflicts when Law Firms Merge


- The conflicts problems arising when a lawyer moves from one firm to another are
compounded when two firms merge
o In a merger all the current clients of each firm become the current clients of
the surviving firm under M.R. 1.7, and the former clients of both firms come
within the purview of M.R. 1.9
- Courts say: notice to affected clients should be given when the negotiations reach the
stage that merger has reasonable likelihood
Remedies against a lawyer in conflicts of interest cases:
- Disqualification of the lawyer
- Sue for malpractice/civil damages (would take care of a portion of damages)
- Sue for forfeiture of fees
- Criminal liability (more rare)
Impacts on a client where there is a conflict of interest:
- Rescission of a transaction that the lawyer did
- Potentially the loss of a claim or a defense, or the reopening of a matter (primarily a
civil matter) automatic new trial
- Client may have to bear the cost of bringing in new counsel and in particular,
recreating or coming up with new work product (may not get the benefit of the work
that was already done, may only get the fees back)
Model Rule 3.2 duty to expedite litigation
Model Rule 1.4, 1.8, and 1.10 know this!

41

CHAPTER 6 WHO IS THE CLIENT?


-

The client-lawyer relationship is between an individual client who is an adult (not a


minor), a person of some experience in ordinary affairs of life (not unsophisticated),
of sound mind (not suffering mental disorder or effects of age or of intoxicants), not
under adverse influence from others (such as spouse or employer), and strong
enough in mind and spirit to interact with reasonable competence with a lawyer in
exercising the authority and responsibility vested in a client (M.R. 1.2)
The lawyer is free of conflicting loyalties (M.R. 1.7 1.10), competent in the matter
involved (M.R. 1.1) and dedicated to doing a good job (M.R. 1.3)
o This means that the lawyer acts to serve the client exclusively (implied by
conflict of interest concept in M.R. 1.7)
Lawyers owe various duties to certain others who are not clients
When a lawyer does something that assists or furthers the interest of
another, it does not follow that the other was a client
In many situations, the live persons with whom the lawyer is
interacting are not clients, or at least not the only clients. In
representation where the client is a corporation or other organization,
the persons with whom the lawyer interacts are not the clients, for the
client is the corporation
The corporation interacts with the lawyer through persons
acting as agents for the invisible corporeal entity
o Therefore, other persons interested in the matter either have no legally
recognized interest or are competent to act for themselves, including
competence to engage a lawyer where appropriate

Individual or Enterprise
- A lawyer is required to communicate and confer with a client, to keep the client
secrets and to abide by the clients decision on whether to accept a settlement offer
- But what about when the lawyer is representing a corporation? Who is considered to
be the client then?
Rule 1.13(a) A lawyer employed or retained by an organization represents the
organization acting through its duly authorized constituents
o You represent the corporation as a whole (not the individual constituents)
Meehan v Hops
361 Pacific 2nd 10
- Corporation Meehan used to work for sued Meehan for breach of duties
- He claimed conflict of interest because his own lawyers were coming after him
- Ps claimed Meehan as the constituent was not the client, the corporation was not
- Therefore, he could be sued
EF Hutton & Co Brown
- Distinguished from the above case
- The lawyers had made an affirmative statement that they were Browns lawyers I
am representing Mr. Brown at the stand
- This gave a reasonable implication/belief that Mr. Brown was represented individually,
and not just the corporation

42

The Organization and its Constituents


In Re Grand Jury Subpoena
Procedu
- Government came to Oldco asking for privileged information
- Lawyers representing Oldco are argued to be personal lawyers as well
ral
- Roe and Moe, the former executives, have the position that the lawyers
History
who represented Oldco were also their personal lawyers, and thus, any
information given was privileged
- Oldco had waived the privilege, but Moe and Roe are claiming that this
info is their matters too
Issue
- What is the extent of attorney-client privilege when there are joint
clients?
Holding
A corporation may unilaterally waive the attorney-client privilege with
respect to any communications made by a corporate officer in his
corporate capacity, notwithstanding the existence of an individual
attorney-client relationship between him and the corporations counsel
Reasoni
- Roe and Moe had given information in their corporate capacity
- They would have still given this information they could not have said no
ng
to the corporation because of their positions in the company their
positions created a fiduciary duty between the constitutes and the
corporation itself
- Court: if you had to give this information anyway you had a
duty to turn it over you can not claim joint-defense
Individual Attorney-Client Privilege Claims
- Attorney-client privilege protects communications made in confidence by
a client to his attorney
-

The privilege only applies to the extent necessary to achieve its


underlying goal of ensuring effective representation through open
communication between lawyer and client
Can only claim attorney-client privilege if the lawyer represented you
individually
Presumption that the attorney only represents the corporate entity, not
the individuals within the corporate sphere, and it is the individuals
burden to dispel this presumption
Roe and Moe can mount a claim of attorney-client privilege only if, and to
the extent that, Lawyer represented them individually the attorney only
represents the corporate entity, not the individuals within the corporate
sphere

The Corporations Right to Waive the Attorney-Client Privilege


- Must remember that the former co-clients were not independent actors,
but rather, corporate officers who owed a fiduciary duty to the
corporation
- The managers interest must yield to the shareholders interest in
disclosure of the privileged materials
- It follows that Roe and Moe may only assert an individual privilege to the
extent that communications regarding individual acts and liabilities are
segregable from discussions about the corporation

43

Commen
ts

In re Bevill TEST: 5 Benchmarks that corporate employees seeking to


assert a personal claim of attorney-client privilege must meet:
1. Must show that they approached counsel for the purpose of seeking
legal advice
2. Must demonstrate that when they approached counsel they made it
clear that they were seeking legal advice in their individual rather
than in their respective capacities
3. Must demonstrate that the counsel saw fit to communicate with them
in their individual capacities, knowing that a possible conflict could
arise
4. Must prove that their conversations with counsel were confidential
5. Must show that the substance of their conversations with counsel did
not concern matters within the company or general affairs of the
company
o If communication specifically focuses upon the individual officers
rights and liabilities, then the 5th prong can be satisfied even
though the general subject matter of the conversation pertains to
matters within the general affairs of the company
The Bevill Test is meant to differentiate between 2 different kinds of
communications between a corporate constituent and counsel for the
corporation:
1. One made in the constituents capacity as an agent of the organization,
made on behalf of the organization
o Recall: Upjohn test determines which communications by
corporate constituents are covered by the corporations attorneyclient privilege
o With respect to these communications, the corporation itself is
empowered to assert or waive the privilege
2. One made in the constituents individual capacity
o Must follow the 5-factour test above to determine when corporate
employees can assert an individual privilege
Joint Representation of Corporations and Their Constituents
- If Moe and Roe had an individual attorney-client privilege to assert (the
issue that is resolved by Bevill), and if Lawyer represented them
concurrently with Oldco under a joint defense agreement, then Moe
and Roe would be empowered to insist that Lawyer not disclose these
privileged communications
o Only to the extent of the communications which involved Roe and
Moes individual rights and responsibilities
Implied Relationships and Miranda Warnings to Corporate
Constituents
- An attorney-client relationship with the constituents of an entity client
that arise by implication
o i.e. corporate constituents disclose confidential info to the
corporations lawyer, under circumstances that suggest a
reasonable belief that the lawyer will keep the info confidential
- Entity-Representation Rule, 1.13
o Requires that lawyers take precautions to prevent a constituent
from mistakenly forming the belief that the lawyer represents the
constituent in her individual capacity

44

o
o

Informally called a corporate Miranda warning or an Upjohn


warning
1.13(f): In dealing with an organizations directors,
officers, employees, or other constituents, a lawyer shall
explain the identity of the client when the lawyer knows or
should know that the organizations interests are adverse
to those of the constituents with whom the lawyer is
dealing

Professional Obligations to Corporations and Corporate Officers


Model Rule 1.13(a) A lawyer employed or retained by an organization represents the
organization acting through its duly authorized constituents
Model Rule 1.13(g) A lawyer representing an organization may also represent a
constituent, as long as the representation does not create a conflict of interest under Rule
1.7
- It may be impossible for the lawyer to remain impartial between jointly represented
clients
- A conflict may arise because of duties of confidentiality owed to each of the joint
clients
-

When conflicts erupt, lawyers frequently err by assuming that they owe duties to
constituents when in fact they should be acting solely in the interests of their
organization client

Individual vs. Corporate Attorney-Client Privilege


- Anything that individuals communicate to the lawyer in connection with their duties
as corporate officers is already deemed to be known by the corporation under the
agency law doctrine of imputed knowledge
- Vicarious-knowledge principal from agency law: if an agent for an organization
communicates with the lawyer for that organization, on a matter concerning the legal
rights and duties of the organization, the information belongs to the principal, not
the agent
Model Rule 1.13(f) in dealing with an organizations directors, officers, employees . . . or
other constituents, a lawyer shall explain the identity of the client when the lawyer knows or
should know that the organizations interests are adverse to those of the constituents with
whom the lawyer is dealing.
Almost Clients
Representing a Closely Held Corporation or Partnership
Fassihi v. Sommers, Schwartz, Silver, Shwartz and Tyler, P.C.
Facts
- In 1973, two doctors (Lopez and plaintiff) each own 50% of X-Ray
company
- In 1975, Dr. Lopez did not want to be associated with plaintiff anymore so
went to attorney for corporation (Epstein) and asked how plaintiff could
be ousted
- Epstein sent letter to plaintiff about termination claiming it was decided
after Board of Directors meeting (even though both doctors were equal
members in the Board)
- Plaintiff went to do his normal shifts at St. Marys but was told that he

45

Issue
Holding

Reasoni
ng

Commen
ts

was not allowed as per prior agreement made between Lopez and
St.Marys by Epstein that plaintiff was never aware of
- Plaintiff states that: defendant had represented both Lopez individually
and the professional corporation without disclosing to him this dual
representation
What duties, if any, does an attorney representing a closely held corporation
have to a 50% owner of the entity, individually?
- Defendant did not have obligation to divulge the existence or contents of
the Lopez-St.Marys contract to plaintiff because Defendants knowledge
of this arose out of a confidential attorney-client relationship between it
and Dr. Lopez
- With respect to communications defendant had with Dr.Lopez while
representing the corporation, as opposed to Lopez personally, plaintiff, as
a member of the control group, is equally entitled to this information
- The attorneys client is the corporation and not the shareholders
- Although no attorney-client relationship exists between plaintiff
and defendant, this does not necessarily mean that defendant
had no fiduciary duty to plaintiff plaintiff was unaware of dual
representation
Garner Doctrine: Good Cause standard: Restatement s.85
- The corporate version of the client-fraud exception
- In a proceeding involving a dispute between an organizational client and
its shareholders, members, or other constituentstoward whom the
directors, officers, or similar personsbear fiduciary responsibilities, the
attorney-client privilege of the organization may be withheldif the
tribunal finds that:
(a) those managing the organization are charged with breach of their
obligations to the shareholders, members or other constituents or
toward the organization itself
(b) the communication occurred prior to the assertion of the charges
and relates directly to those charges; and
(c) the need of the requesting party to discoveris sufficiently
compelling and the threat to confidentiality sufficiently confined to
justify setting the privilege aside

Skarbrevik: Counsel for Close Corporation with No Fiduciary Duties to Shareholder


- A lawyer for a four-shareholder corporation owed no legal duty to one of the four
shareholders and thus could not be sued by that person for negligence
- Three shareholders were not happy with the fourth and wanted to buy him out but
instead were required to use their money to keep the corporation afloat. They then
came up with a plan to dilute the value of Skarbreviks interest (with the assistance
of corporate counsel) but before additional stock could be offered, it must first be
offered to Skarbrevik
- Lawyer told others that Skarbrevik had to be given notice, and knew that notice was
not given but still continued on with assisting plan to dilute Skarbrevik
o In this case there was no basis for plaintiff to place special trust or confidence
in corporate counsel, or for him to rely on corporate counsel to act in his best
interest in relation to the corporation or the other shareholders
Co-Client Approach to Shareholders of Closely Held Corporations

46

In closely held corporations where the operator of the corporation either owns or
controls the stock in such a manner that it is reasonable to assume that there is no
real reason for him to differentiate in his mind between his own and corporate
interests, the lawyer for the corporate owes that person the same duty not to
represent conflicting interests that she would owe a client

Partnerships
- A lawyer representing a limited partnership, upon discovering that a general partner
has committed acts adversely affecting the interests of limited partners, may disclose
those facts to the limited partners so that they will be able to take steps to protect
their interests
Representing a Fiduciary
Fickett v. Superior Court of Pima County
Facts
- Schwager was appointed guardian of Mrs. Styers estate and sold most of
her common stock to make a building for his benefit
- This proceeding was brought against the law firm which had represented
Shwager as guardian
- Claim that petitioner was negligent in failing to discover that the conservator
had been engaged in an ongoing scheme to defraud the estate. Petitioner
argues that, absent a showing of fraud or collusion, he had no duty to the
conservators ward. The lower court rejected this argument, and Petitioner
now appeals.
Issue
Holding
Ratio
Reasoni
ng

May a conservators attorney be held liable for failure to detect misappropriation


of an estate assets?
Yes
Attorneys may incur liability to non-clients in many situations in which it is
foreseeable that their actions might harm them.
- When an attorney undertakes to represent the guardian of an
incompetent, he assumes a relationship not only with the guardian but
also with the ward
- In fact, the wards interest overshadow those of the guardian
- This is because of the relationship between the attorney and the intended
beneficiary; public policy requires that the attorney exercise his position
of trust and superior knowledge responsibility so as not to affect
adversely persons whose rights and interests are certain and foreseeable
Attorney not liable to one not in privity of contract on 2 grounds:
- To allow such liability would deprive parties to contract of control of their
agreement
- Duty to general public would impose a potential burden of liability on
contracting parties.
- Many factors impacting privity and third party duties argument: extent to
which transaction intended to affect party, foreseeability of harm, degree
of certainty they suffered injury
- Public policy permits imposition of duty in these circumstances attorney
owes duty to ward and to guardian

Commen
ts

Attorneys may, as a matter of public policy, be held liable to non-clients


after a number of factors are considered, including the extent to which
the transaction was designed to affect the plaintiff, the foreseeability of

47

harm to him, the degree of certainty that plaintiff would suffer injury, the
connection between defendants conduct and plaintiffs injuries suffered,
and the policy of preventing future harm.
When an attorney represents the guardian of an incompetent, he
assumes a relationship not only with the guardian, but also with the
ward.
Model Rule 1.14 Diminished capacity

KNOW RULE 1.18


- A prospective client is entitled to certain duties (i.e. someone who
comes in for a consultation but never ultimately hires you)
- Must avoid conflicts of interest
Attorney-Client Privilege for a Trustee
- Restatement s.83
o In a proceeding in which a trustee of an express trust or similar fiduciary is
charged with breach of fiduciary duties by a beneficiary, a communication
otherwise covered by attorney-client privilege is nonetheless not privilege if
the communication:
(a) Is relevant to the claim breached; and
(b) Was between the trustee and a lawyerwho was retained to
advise the trustee concerning administration of the trust
Hazard: Triangular Lawyer Relationships
- Examines the nature of a lawyers responsibilities where the lawyers client has a
special legal relationship with another party that modifies the lawyers normal
professional responsibilities the triangular relationship
- First type: client in a fiduciary relationship to a third party
o i.e. lawyer representing a guardian in matters relating to the guardians
responsibilities
to a ward
Guardia
Lawye
Lawyer Guardian Ward
n
r
Other:

Ward
-

Lawyer General Partner


Partnership

Second type: involves a third party who owed fiduciary duties to the lawyers
client, and the third party rather than the client is the one with whom the
lawyer deals ordinarily
o i.e. a lawyer who represents a corporation but who, in the ordinary course of
professional service, deals with the corporations officers, directors, and other
employees.
Corporation
Lawyer Corporation Officer
Lawye
r
Other:
Officer

Lawyer Partnership General


Partner

In the lawyer-guardian-ward relationship the ward is the dependent


person and the oblige of the guardian, but the guardian is the
dependent person and the primary oblige of the lawyer

48

In the lawyer-corporation-officer triangular relationship, the corporation


is the dependent entity and the oblige of both the lawyer and the
corporate officer

Three Possible Relationships


1. Client
o The lawyers primary duty to the client is loyalty and the relationship may be
analogized to that of a limited-purpose friendship
2. Courts
o The lawyer owes the court only a minimal duty of diligence and candor (more
structured relationship)
3. Third party
o Against a lawyer, a third party is entitled to the protection of the criminal law
and the law of fraud

49

CHAPTER 7 LITIGATION

Client Perjury
- A lawyer who lies to a third party while facilitating a fraudulent transaction may be
liable under civil or criminal law for fraud
- The lawyer who acts recklessly in handling a transaction may also be liable for
criminal or civil fraud, and one who acts carelessly may be liable for negligent
misrepresentation
o In the civil context, client perjury cannot be done (cannot advise or allow); in
the criminal context, there are unique aspects (narrative approach), but
generally you cant do it as well
o Witness perjury again there is no issue here whether civil or criminal
o The rule with regard to perjury issues is MODEL RULE 3.3 (READ ALL
especially Comment 6, 10)
o The four Rs
Remonstrate
Talk with the client, if necessary asking for a recess in the
proceedings, in order to do so you give the advice that the
comment is perjury (tell client it cannot be done, as it is illegal)
Rectify
Rectify the perjury by advising the client correct the false
statement/record
Resign/Remove
Resignation from the representation if the client refuses to
rectify (if possible)
When you file your appearance in a matter, there is a rule that
says that you are the attorney of record in the case and it says
you cannot get out of the case unless and until the court
provides an order allowing you to resign this is why it is resign
IF POSSIBLE this is only a request because there is always
the chance the court will not allow you to resign
Remove the statement
Reveal
Even if you are allowed to resign, you may still have the duty to
reveal difficulty comes if you are not allowed to resign as
issues of confidentiality may occur
If you are able to resign then by revealing, you are not being
disloyal but you are not usually supposed to break the loyalty
of current clients
Reveal the falsity to the court and counsel
Perjury and the Model Rules
- A lawyer who knows a witness is lying has both prospective duties, to prevent the
client or non-client witness from testifying falsely, and retrospective duties, to take
remedial measures if the lawyer learns later that the testimony was false
- Actual knowledge (M.R. 1.0(f)) applies but it may be inferred from circumstances,
and courts are often unwilling to believe lawyers who say that they could not possibly
have known that a patently unbelievable story was false. (Can be found through the
use of circumstantial evidenceto what degree though is not known).
-

50

Rule 3.3
o A lawyer must disclose to the court all facts that would otherwise assist a fraudulent act
o Sets up two categories:
Retrospective: a lawyer shall not knowingly fail to correct a false statement or fact of
law
Prospective: shall not make a false statement or fact of law to a tribunal

Model Rule 3.3(a)(3) prohibits a lawyer from offering evidence that the lawyer knows to
be false if concerning evidence that is material [likely to impact the outcome of
the proceeding, per 3.3(a)(1)].
- Lawyer must take reasonable remedial measures up to and including disclosure of
the perjury to the tribunal (if they are surprised by perjury on the stand or later
become aware of it)
Model Rule 3.3(a)(1) requires disclosure of false statements of material facts
Model Rule 3.3(b) requires the lawyer to take reasonable remedial measure if they have
the knowledge that someone has or is engaged in any sort of criminal or fraudulent conduct
related to the proceeding such as bribery, jury tampering, or destruction or concealment of
documents or evidence
Model Rule 3.3(c) limited to where the lawyer learns of the falsity prior to the conclusions
of the proceeding (once the case is over it does not need to be divulged)
Model Rule 4.1 fraud on non-tribunal third parties (it requires lawyers to disclose material
facts where necessary to avoid assisting in criminal or fraudulent act by a client, UNLESS the
information is covered by M.R. 1.6
Model Rule 5.2(b) A subordinate lawyer may, however, act in accordance with his or her
supervising lawyers reasonable resolution of an arguable question of professional duty,
If a lawyer is surprised by perjury, or subsequently comes to learn that a witness
testimony was false, the lawyer is required to correct the record
o Process
Remonstrate the Client Confidentially
Advise Them to Correct the Record
Seeking Withdrawal from Representation (M.R. 1.16(c)
tribunal may not permit)
o If the less drastic measures do not work, the lawyer must disclose the perjury
if it is the only means of rectifying the perjury, even if the information would
otherwise be protected by 1.6 (duty of confidentiality)

Perjury in Civil Cases


- Much more occurs with civil cases than with criminal cases
United States v. Shaffer Equipment Co.
Facts
- EPA tried to recover $5M for waste dumping from Shaffer Equipment Co.
- The EPA on-site coordinator (Caron) of the cleanup site had
misrepresented his credentials
o This was material because Caron was responsible for making the
administrative record that was used to determine liability
o Nobody argued that Carons analysis was wrong, just that he
didnt have the academic degrees he claimed to have had
- The EPA failed to disclose this fact to Shaffer or the Court until after
discovery and the deadline for filing summary judgment motions
o EPA attorneys were aware of Carons deception but continued to
press the cases C was involved in
o They decided not to disclose his credentials
- EPA had violated Rule 3.3 they found there was an effort to cover up a

51

long history of fraud


Rule 3.3.
o A lawyer must disclose to the court all facts that would otherwise
assist a fraudulent act
o Sets up two categories:
Retrospective: a lawyer shall not knowingly fail to
correct a false statement or fact of law
Prospective: shall not make a false statement or
fact of law to a tribunal
Whether the district court had jurisdiction to dismiss the action based on Snyder
and Hutchinson being found in violation of Rule 3.3 Prof. Conduct (candor to the
tribunal).
Federal Appellate Court affirmed that the government attorneys breached their
duty of candor under the rule 3.3. The Court however reversed the order of
dismissal and remanded for consideration of a lesser sanction.
Rule 3.3 says that a lawyer must disclose to the court all facts that would
otherwise assist a fraudulent act.
The Trial Court found that Snyder and Hutchins had acted egregiously
because they both knew about Caron's deception and obstructed discovery
requests which could have revealed the deception to Schaffer.
o The Government argued that Shaffer had failed to establish all
three elements necessary to show a Rule 3.3 violation:
That the attorneys had actual knowledge of the
misrepresentation. Not only did Caron admit the lie, the
additional investigations turned up more discrepancies in
the testimony.
That Caron's credentials were a material fact to the
case.Had the credential not been there the ability to
approve actions on site would not be allowed, so it is
material in determining credibility.
That the attorneys' conduct assisted EPA in a fraudulent
act.
- The Federal Appellate Court affirmed the fact that the EPA had violated
Rule 3.3, but reversed the Trial Court's decision to dismiss the case.
o The Appellate Court remanded the case back to the Trial Court to
determine a sanction short of dismissal.
The Appellate Court noted that dismissal is a legitimate
sanction for a Rule 3.3 violation.
However, in this case dismissal would provide relief beyond
the harm caused by the violation and would frustrate
resolution on the merits of the case.
o The Appellate Court found that while all attorneys have a duty of
confidentiality to their client and must advocate their client's
position vigorously, they also have a duty to assure the integrity
of the process and guard against the corruption that justice will be
dispensed on an act of deceit. (Integrity trumps Duty)
o The EPA's main argument was that Caron's credentials were not
material to the case. But the Court found that evidence showed
that Snyder admitted that the fact was material in internal
-

Issue

Holding

Reasoni
ng

52

Commen
ts

memos.
They also violated Rule 3.4(a), by obstructing Schaffers' access to
information via discovery (bad faith between opposing counsel).
The general duty of candor includes that of informing the Court
of any development within the case that may conceivably affect
the outcome of the litigationduty broader than 3.3 owed as
officers of the court.

KNOW:
- Perjury comes up in civil cases, not just criminal
- There is a difference between retrospective and prospective
- Courts will look at these issues in ways that dont just have to do
with zealous advocacy

Perjury in Criminal Cases


Nix v. Whiteside
Facts
- defendant in criminal proceeding petitioned for a writ of habeas corpus
alleging that he had been denied effective assistance of counsel and the
right to present a defense because his defense counsel did not allow him
to commit perjury
o counsel refused to let him say he saw a gun when he stabbed his
victim where he admitted he saw something metallic
Issue
Whether the sixth amendment right of a criminal defendant to assistance of
counsel is violated when an attorney refuses to cooperate with the defendant in
presenting perjured testimony at his trial?
Holding
Not violated. Lawyers representation fell within accepted standards of
professional conduct and the range of reasonable professional conduct
acceptable under Strickland
Ratio
Reasoni
2 Prongs of Strickland Test
ng
To obtain relief by way of federal habeas corpus on a claim of deprivation of
effective assistance of counsel under the sixth amendment, the movant must
establish:
- Behaviour of attorney fell below the norm
- There was prejudice (a clear conflict of interest between a lawyer and
client)
o Exception to prejudice:
In this case there was a conflict between lawyers
responsibility to the client and responsibility to the court.
Under Strickland, if you encounter a conflict, you do not
have to worry about the prejudice
Thus, when the lawyer has a conflict of interest, the D will
always get a new trial
o It was decided that an attorney cannot be conflicted if they are
attempting to do their professional responsibility with a
constitutional right
-

In this case, the having seen the gun was a false statement
This was not the finding of the Supreme Court (trial court)
o The Supreme Court was only dealing with the situation that the

53

testimony was not false


All Ds have a Constitutional right to take the stand
No D enjoys the right to testify falsely thus, cannot be an
interference
There is no prejudice that will be presumed when a D is not
entitled to testify falsely
This prejudice element is important, especially in
the competency area i.e. ineffective assistance of
counsel
-

Point on where the concurrence and the majority split:


o Burger tells us that disclosure of any false testimony is required
this is the standard
The Model rules need to be applied to the Iowa situation.
Iowa did not have Model Rules they were a Code state
and their code was standard to the 1974 Model Code,
which had a provision that said you have a duty to disclose
client perjury unless the information related came as a
result of an attorney-client protected communication
o The concurrence takes issue with this
They state that the US Supreme Court is not the arbiter of
ethics rules
The ABAs statement under 3.3 is the national rule in
regard to these issues

Is Burger right, that disclosure is mandatory?


- Start with Model Code Rule 3.3 (Candor toward tribunal)
o Do we have a conflict between our duty of confidentiality and our
duty of disclosure?
- Look at ABA Rule 1.6 (Confidentiality of Information)
o Look at the COMMENT
Comment 15 As Burger stated, Rule 3.3 in fact
does require disclosure
Thus, under this comment, Rule 3.3 > 1.6

Commen
ts

The Supreme Court of Iowa affirmed the conviction and held that the
right to have counsel present all appropriate defenses does not
extend to perjury, an attorneys duty to a client does not extend to
assisting a client in committing perjury, and the attorneys actions in the
instance were required.
The court reasoned that an intent to commit perjury when communicated
to counsel does not alter a defendants right to effective assistance of
counsel.
Thus, the attorneys admonition to Whiteside constituted a threat to
violate the attorneys duty to preserve client privilege. Therefore, the
Court of Appeals found that the threatened violation of client confidences
breaches the standards of effective representation laid out in Strickland
v. Washington. They say this was a conflict of interest as he was more
concerned with his own situation than the clients situation.
Mistrial issues are extremely worrisome as there is no double jeopardy
allowed as a constitutional right. The disclosure or withdrawal request
may create an extreme issue in terms of avenues for mistrials.

54

Rule 3.3 Comment 10:


Remedial Measures
- Ways for the Court to correct perjury
3.3(a)(3)
- Michigan lawyers are subject to ABA and certain court rules
- Duty to report perjury ends once proceeding ends
Court Truth and Real Truth
- When a lawyer knows that evidence is false
o The lawyers suspicions are not enough, it usually is a result of the clients
communications
o Wisconsin Courtwithout a direct and unambiguous statement that the client
intends to falsify testimony, the knowledge element of M.R. 3.3(a) has not been
satisfied (State v. McDowell)
o New York holds to a lower standard where they permit criminal defense attorneys
to infer knowledge of contemplated perjury from the patently unbelievable nature
of their clients stories. (People v. DePallo)
o If a lawyer practices the theory dont ask so they can never truly know if the
evidence is perjurious or not, may be in violation of the ethical rules
Model Rule 3.3(a)
Government Use of Perjured Testimony to Obtain a Conviction
- If the government knowingly used perjured testimony to secure a conviction, the
defendant is entitled to a new trial if the false testimony could . . . in any reasonable
likelihood have affected the judgment of the jury. . . . (i.e. change the outcome)
What Should a Criminal Defense Lawyer Do?
- If a lawyer cannot dissuade the defendant from committing perjury, the lawyer
should have the defendant tell her story in a narrative without further questioning by
counsel must also put on record that defendant is taking the stand against the
lawyers advice
- Requesting to withdraw here is almost always deprivation of a fair trial, as it is an
announcement to the trier of fact that the lawyer believes there to be perjury within
the testimony
The Narrative Approach
- M.R. 3.3 cmt [7]
o A lawyers obligations regarding Rule 3.3 are subordinate to other law requiring
the narrative approach
- Some states have adopted the narrative approach, and some ethics rules regarding it as
well
- The Supreme Court has recognized that it is the defendants constitutional right to testify
in criminal cases, but repeated the Whiteside statement that this does not include the
right to testify falsely.
- The witness is allowed to testify with whatever they want to saythe lawyer cannot aid
the testimony so they are only asked What would you like to tell the jury (as opposed
to normal testimony where all testimony is to be obtained through questions, so there is
a valid objection for narrative). The prosecutor is instructed to refrain from objecting as
the testimony is narrative in nature.
- Not a violation to follow the local custom to this if it is allowed, but the ABA does not
prefer it
o Questions will be assuming that the ABA rules apply

55

Model Rule 3.3(a) shall not knowinglyfail to discloseoffer evidence the lawyer knows to
be false
- You can put the client up, and ask them questions, cannot presume they will perjure
themselvesthe rules are not triggered until the testimony comes out you cannot have
the necessary knowledge, the duty does kick in once it does come out and you know.
- Have to ask for a break somehow to allow from remonstrating, and rectification
May a Defense Lawyer Inform the Trial Judge that the Defendant Has Committed Perjury?
- The intent to commit a crime is not a protected confidence or secret [DR 4-101(C)(3)]
and could be revealed to the trial judge in order to prevent the clients intended
perjury (People v. DePallo)
Monroe H. Freedman Perjury: The Lawyers Trilemma
Is it ever proper for a lawyer to present perjured testimony?
o The three conflicting obligations of the lawyer in the adversary
system are:
(1) To learn everything the client knows about the case (know they lie
they might tell);
(2) To hold in strictest confidence what the client reveals
(Confidentiality = cannot disclose it); and
(3) To act with candor toward the tribunal (Candor = must disclose it)
(4) Campbell: duty to not violate the law it is still a crime to
commit perjury; thus the lawyer is making himself part of a
crime; thus, even if the Rules somehow permit what Freedman
is talking about, the law does not (the first three would ensure
that you would recognize perjury and so it would be aiding and
abetting in a criminal act)

United States v. Gellene


Facts
- Gellene was a partner in a firm in a bankruptcy suit
- He had a conflict of interest with opposing party and put up a Chinese
wall
- Conflict with 1.7(a)(2) issue raised by JNL (creditor who was not
represented by Gallene)
o Are the creditors actually clientswill court consider them a
client
- JNL knew something was wrong was upset about the Milbank firm
- JNL wanted Bucyrus to sue Salovaara
Issue
Whether the defendants abuse of his role as attorney for the debtor significantly
facilitated the commission of the perjury
Holding
Yes, the defendants abuse of his role as attorney for the debtor significantly
facilitated the commission of the perjury (false declaration outright excluded
South Street)
Ratio
Reasoni
- Knowingly and fraudulently made false declarations under oath on two
occasions: (1) when he filed the initial incomplete disclosure statement
ng
and (2) when he filed a second incomplete disclosure statement in
connection with the hearing on other allegations
- Fraudulent if known to be untrue and made with intent to

56

Commen
ts

deceive Gellene claims he did not have the intent to defraud (which is
different)
To deceive is to cause to believe the false or to mislead; to defraud is to
deprive of some right, interest or property by deceit this is not accepted
by the court
Also Gellene claimed he was a member of the New York bar, and
deceived people for nine years which added evidence to the
governments argument that Gellene did have the intent to deceive
(allowed to use such bad acts, as Gellene argued that he did not have the
required intent for fraud)
M1: Bacyrus v. South Street
M2: South Street v. Trustee
South Street purchased the equipment owned by Bacyrus pursuant to a
leaseback agreement in order to help them out
Model Rule 1.7(a)(2) this is an issue of a conflict of interest with a current
client, and an argument of direct diversity (between the current clients)
but it would be hard to release the money however could argue that the
Trustee wants to evenly distribute the money, if it was Model Rule 1.9
(former client) may be easier to get around as there is no longer loyalty
to the client.

Conflicts issue:
- Rule 1.7
- What is the conflict of interest?
o When you are representing a bankruptcy filing for one company
and the companys creditors
o There is a duty to treat all creditors fairly
- Who raises the conflict issue?
o JNL a creditor of the company
- Gellene could have disclosed everything and probably still could have
been the attorney for the bankruptcy
- How would Gellene avoid the conflict of interest?
o Consent from the client
1.7(2)
1.7(B)
See if he can get a knowing and informed waiver in writing
o Why didnt he do this?
NOW, lets look at this case from the criminal side
o Gellenes false statements
o He had a history of lying
- Gellene had a higher duty of candour than the other case
o He was submitting affidavits because he wanted to be the
attorney for the trustee
- WHAT GELLENE DOES WRONG:
o He acts as an advocate on his own behalf while filling out these
forms
o He takes an approach toward the information that it not consistent
with the idea that he is going to be neutral and fair to all parties
involved in the bankruptcy
o He has become more of a partisan than an adjudicator
-

A trustee with full knowledge would realize it would cost more to bring
someone else they would have waived HAD HE DISCLOSED; but he

57

didnt
-

Milbank firm had to forfeit their fees they were hired on the basis of
fraud

What if the firm didnt know?


o Rule 3.1 concept of imputed knowledge

Bronston and DeZarn


Gellene relied on the Bronston case in terms of support for his argument for his remarks on
the stand being literally true yet it is appositive the Gellene case (at issue is a witness
orchestrates his own false evidence under direct examination through the planned questions
of his own attorney, rather than the adversary examination of a witness in Bronston).
-

Bronston, USSC: defendants alleged perjury occurred in a bankruptcy proceeding,


and when asked about Swiss Bank accounts he testified that he currently had none,
and later that he had never had one.
o He did not have one at this time, but had in the past
o Supreme Court unanimously reversed the conviction for perjury on the ground
that his non-responsive answer was literally true, albeit misleading
DeZarn, 6th Circuit: distinguished Bronston and upheld a perjury conviction
based on materially misleading answers, where he testified falsely as the
prosecutor used the wrong date of the party in his questionthe perjury
conviction was upheld as a defendant may be found guilty of perjury if a
jury could find beyond a reasonable doubt from the evidence presented
that the defendant knew what the question meant and gave knowingly
untruthful and materially misleading answers in response

Perjury, False Swearing and Subordination


- Any false statement under oath may constitute perjury, as long as it was an
intentional falsehood (i.e. a lie) and concerned something that was material
to the proceeding in which the lie was offered
o The falsehood must be knowingly offered under oath, i.e., no showing of
willfulness is required
o If the defendant has made two contradictory and irreconcilable statements
under oath, the prosecution need not prove which of the two is false
Obstruction of Justice and the Anderson Case
- Whoever corruptly . . . endeavors to influence, intimidate, or impede any . . .
officer . . . of any court of the United States . . . in the discharge of his duty . . . or
corruptly . . . influences, obstructs or impedes, or endeavors to influence, obstruct, or
impede, the due administration of justice, shall be fined . . . or imprisoned . . . or both
- Conduct commonly treated as obstruction of justice includes: attempting to alter or
prevent the testimony of a witness, interfering with a grand jury investigation and
destroying evidence sought by a court or grand jury
** dont need to know perjury or obstruction of justice statutes but NEED TO KNOW Gellene
case! **
Anderson Case
- Illinois lawyer telling people in Texas what to do to meet their legal obligations
- Rule 5.5 deals with unauthorized practice of law and sets the limit

58

Conflict of Interest Hypo: Lawyer as a Witness


A = driver
B, c, d = passengers
Car accident occurs in front of L
L is a lawyer
L is your client
A-D Come to L and ask them to represent them in a lawsuit based on the accident
Can L take on the representation? L asks you?
What advice do you give L? What rules do you review with L?
-

This is a conflict of interest


o Conflict between lawyer and client
*Rule 3.7: Lawyer as a Witness
o Does not disqualify the lawyer just because they saw the action they may be
limited because they may be called by the other side depends on whether or
not there are other ways to present the evidence or if there is a way the
parties can stipulate
*Rule 1.7: Conflict of Interests: Current Clients
o If they lawyer saw something unfavourable to their clients (1.7(b))
*Conflict of interest bw A, B, C, D
Rule 1.18: Duties to Prospective Clients
o Not necessarily
Rule 1.1: Competence
o Maybe 1 point for this
Rule 3.4: Fairness to opposing party and counsel
o Maybe, if you had Rule 1.7 as well
Do not worry about Rule 1.16 (Declining or Terminating Representation) because the
lawyer has not yet taken on the client
Rule 1.4: Communications
Rule 1.2
Rule 8.4(a): Misconduct to Violate a Rule or to Attempt to Violate a Rule
o PUT IN FIRST QUESTION

59

CHAPTER 8 LAWYER-CLIENT RELATIONSHIP


Main question: how the law and ethics of lawyering shape the conditions in which the lawyer
pursues the clients legitimate goals
-

Fees
-

Inquiry into the lawyer-client relationship has 3 dimensions:


o Normative Dimension how should the lawyer-client relationship be
structured?
Market or Contractarian Model
Views the lawyer-client relationship as fundamentally a
consensual and mutually beneficial exchange
o i.e. client receives services conforming to contract terms
and market standards; lawyer receives payment for
services rendered
Fiduciary Model
Views the lawyer-client relationship as fundamentally an agency
relationship, with the client dependent upon her lawyers skill
and knowledge. A client must be able to trust her lawyer to
provide loyal, competent and diligent services at a reasonable
cost
Regulatory or Public Utility Model
Views lawyers as quasi-public officials performing important
public functions, which greatly affect parties outside the lawyerclient relationship
o Empirical Dimension how do lawyers and clients actually behave in
various sectors of practice?
o Legal Dimension how do controlling principles of agency law, contract law
and legal ethics regulate the relationship?
In reality, the 4 principal qualities clients seek in their lawyer are commitment,
integrity, competence and a fair and affordable fee

Michigan uses clearly excessive and is defined as unreasonable, and the ABA is
unreasonable fees
Understand what the abas view is on fees good question for part 1-2 of exam
o BASIC RULE: There are only two types of money in this world: (1) earned and
(2) unearned
Unearned money HAS to be held in trust the vehicle for that in
almost every state is IOLTA account (interest on lawyer trust account)
This is not taxable money because it is not the lawyers actual
money, but the clients until it has become transferred into
earned funds it is pretty much a client-trust account
A separate account from any of the lawyers controlled interests
or assets
Earned money, with very few exceptions, can never be in an IOLTA
if you keep any earned money in an IOLTA, or put earned money
inside this is co-mingling of funds and is a violation (also
considered as tax evasion)

60

Conversion (converting money) is taking money out of the IOLTA for a reason
other than representation. Converting somebodys money or misappropriating
it is also a violation.
Cardinal sin to misusing money and abusing the trust of the client in
taking money that should not have been taken
Make sure that all money that comes into your possession as a lawyer is either
specified as earned or unearned; this must be understood by you and by the client
o Once the fee is earned, then you have to promptly take that money out of the
IOLTA account (reasonable period of time is allowed)
o In a true IOLTA any interest that is accrued goes to whoever is named as the
beneficiary in the court rules (usually this is the Bar)
With regard to fees, there is such a thing as an advanced fee which is unearned
money this must go into the trust account
There is another type of fee called earned advanced fees (earned on receipt): (2
types)
o Classic retainer:
The client says I will pay you x amount of dollars just to be able to call
on you to be my lawyer at another time;
Buying the right to have the lawyer do work at some later time and will
pay the lawyer later to do this actual work as well
There are two reasons to do this: makes somebody available to
your side; makes someone unavailable for the other side (due
to conflict of interest [Rule 1.7]).
o Non-refundable retainer or minimum fee (more controversial and differs
by jurisdiction)
The State of New York prohibits the use of the word non-refundable in a
fee agreement between a client and lawyer
All fee agreements must be in writing because if you want to
collect on the fee agreement you have to sue your client on an
unpaid bill and you cannot do this unless you have a written
agreement
Michigan has taken the opposite view: non-refundable is a way
of saying that money is earned upon receipt
It is a word that HAS to be in all agreements if it is nonrefundable that means it is being earned by the lawyer upon
receiptit should be bold and in all CAPS
There is no one theory that explains how you will have a nonrefundable fee
Advanced payment is presumed to be a special retainer (a deposit against future
services) unless the client and lawyer otherwise agree
When a fee agreement is ambiguous, courts construe the agreement against the
lawyer who drafted it (contra preferentum)
According to professor: earned and unearned is DIFFERENT than
refundable/non-refundable
Lawyer-client relationships is basically a contractual relationship of fee for service
o The lawyers primary obligation is to provide competent representation to
advance the clients lawful goals and the clients primary obligation is to pay
the lawyers fee
o

Four types of fee-arrangements, separately or in combination, predominate in a


lawyer-client relationship
o (1) a flat fee for a particular legal matter (e.g. $1500 for a simple uncontested
divorce)

61

o
o
o

(2) an hourly rate fee (e.g. $300 per hour for work on a particular matter
times the number of hours)
(3) a proportional fee (e.g. handling a real estate transaction for a percentage
of the purchase price)
(4) a contingent fee (e.g. a fee of $10000 to be paid if a particular result is
obtained)

Model Rule 1.5 addresses the regulation of fees


(a) A lawyer shall not make an agreement for, charge, or collect an unreasonable fee or
an unreasonable amount for expenses (lists 8 non-exclusive factors relevant in
determining the reasonableness of a fee)
Amount of the Fee
In Re Fordham
Facts
March 1989, Timothy, aged 21, charged with operating under the influence of
alcohol and operating a motor vehicle after suspension. A partially filled bottle of
vodka was found in the unregistered vehicle. Timothys father consulted three
lawyers who offered to represent Timothy for fees between $3000 - $10000.
Father went to home of Fordham to service an alarm system and conveyed his
sons predicament and Mrs. Fordham told him that her husband could represent
Timothy. Fordham had never done a case like this but said he was a good
lawyer who operated under an hourly rate of $200/hr. After all the work was
done, court found Timothy not guilty and Fordham sent monthly bill
arrangements totalling $50022.25 (227 hours of billed time). Father paid $10000
and signed promissory note evidencing his debt to Fordham.

Issue
Holding

Ratio
Reasoni

The Bar Counsel charged Mr. Fordham for charging an excessive fee from his
client under Disciplinary Rule 2-106. The hearing committee after going through
the hearing, concluded that the fee charged by Mr. Fordham was not
substantially in excess of a reasonable fee and this opinion was further accepted
by the board of overseers. The Bar counsel appealed to the Supreme Judicial
Court of Massachusetts requesting the court to affirm that the fee charged by
Mr. Fordham is excessive.
- Was Respondents fee in this matter clearly excessive?
Yes
- Yes. Although it was reasonable of Respondent in this case to take the
case even if he was not experienced in the relevant law, it was not
reasonable of him to expect the client to pay for his education in this
field of law and then charge him nearly ten times more than an
experienced attorney would have charged.
- Although the client consented to the terms presented by Respondent,
Respondent never gave him a proper estimate of how much the case
might cost and he had no reason to believe that the total bill would be
anything approaching $50,000-especially since the average case in this
area bills about 30 hours.
- His work was not novel enough even though it was creative to file a
motion to dismiss the breathalyzer tests that ultimately seemed to have
led to the win.
-

in a discipline case, the burden of proof is on the person accusedi.e. the

62

ng
-

Commen
ts

lawyer has to prove he met the requirements of the Rules


can have a contract that survives civil suit but does not meet the ethics
requirements

Factors to look at under DR 2-106:


- Examining the time and labor required, the novelty and difficulty of the
questions involved, and the skill requisite to perform the legal service
properly
- Need to look to the novelty and difficulty of the questions involved,
and the skill requisite to perform the legal service properly
- Fordham: did what he had to do to pursue the case to trial
- Timothy: the inexperience in criminal defense work and OUI
cases in particular cannot justify the extraordinarily high fee
- The fee charged must be comparable to the fee customarily charged in
the locality for similar legal services
- Fordham: client entered into agreement knowing of
inexperience and knowing he could get other experienced
lawyers for a cheaper price
- Timothy: although agreeing to hourly rate, had no idea it would
take that many hours and was not given a reasonable estimate
of how much the total fee would be
Unlike the Bushman case (where we have a very sympathetic client trying to get
custody) we have a relatively significant and wealthy client. Mr. Clark who met
Fordhams wife, he owned two unmortgaged race horse farms; he also had his
own independent high end business and managed his mothers wealth.
Rule 1.5 Becomes the issues in both the contract and discipline case
Lawyer has the burden to prove (bc lawyer is trying to enforce the K):
(a) A lawyer shall not make an agreement for, charge, or collect an
unreasonable fee or an unreasonable amount for expenses. The factors to be
considered in determining the reasonableness of a fee include the following:
(1) the time and labor required, the novelty and difficulty of the questions
involved, and the skill requisite to perform the legal service properly;
(2) the likelihood, if apparent to the client, that the acceptance of the particular
employment will preclude other employment by the lawyer;
(3) the fee customarily charged in the locality for similar legal services;
(4) the amount involved and the results obtained;
(5) the time limitations imposed by the client or by the circumstances;
(6) the nature and length of the professional relationship with the client;
(7) the experience, reputation, and ability of the lawyer or lawyers performing
the services; and
(8) whether the fee is fixed or contingent.
- Most people would agree 1.8(a) does not apply
- Make sure the deal is fair
- Tell the other side that they should have or do have the opportunity
to use independent counsel
- Have to disclose all of the terms in writing
- Generally speaking, fee agreements are not covered by this above rule;
most obviously because, it is not a client until the document is signed
- 1.5(a) allows a lawyer to charge whatever they want as long as it is not
excessive

63

Could also potentially look at:


- 1.8(a)midstream rule about finances
o redoing a fee in the middle of a case is a business transaction
o but at the outset is not a business transaction
o fair does not equal unreasonable
neither term is defined but lawyers duty is heightened to
not just unreasonable but also not unfair
- 1.8(i)
o lawyers cant acquire proprietary interest
-

Due process is awarded to lawyers in disciplinary cases

Bushman v. State Bar of California


Facts
Attorney Bushman was retained by a 16-year-old woman and her parents to
represent her in divorce and custody proceedings. The fee agreement included
a $5,000 retainer and a rate of $60/hour. His client won her custody suit, and
the court ordered her former husband to pay reasonable attorneys fees and
costs of $360. (Bushman had not informed the court of the previous fee
agreement.) Bushman than submitted his bill of over $2,800, claiming that he
had worked over 100 hours on the case-even though his opponent had spent
only a little more than five hours.
Issue
Was Bushmans fee excessive under these circumstances?
Held
Yes. Bushmans bill was completely unreasonable in this context. Suspended
for one year.
Under the circumstances, the fee charged by Bushman was so exorbitant and
wholly disproportionate to the services rendered to the defendants as to shock
the conscience.
Know:
Model Rule 1.4(a) and (b)
Model Rule 1.8(b)can a lawyer use the info to the lawyers benefit? No rule suggesting that
you cannot
Model Rule 1.8(a) may be established whether or not the client has suffered an economic
loss
Model Rule 1.8(i)
Model Rule 1.8(j)
Model Rule 1.8(k)
No Model Rule 1.8(h)
Judicial Control of Fees
- Lawyer may sue the client to recover a fee, or the client may sue to recover fees
already paid
- Courts regulate fees in a number of situations, including where courts act in a
supervisory capacity, as in class actions, bankruptcy, probate, and cases involving
minors, etc.
- Courts jurisdiction includes authority to review or determine the lawyers fee

64

Non-Refundable Retainer Fees


- Lawyers often use the term retainer to refer to the variety of fee arrangements
o Special retainer is an advance payment for performance of specific work to
be charged against it considered the property of the client until earned by
the lawyer
o General retainer or engagement fee normally refers to an advance payment
by a client that is deemed earned when received whether to not the client
subsequently calls on the lawyer to perform specific work
- An advance payment is presumed to be a special retainer, that is a deposit against
future services, unless the client and lawyer otherwise agree
o Hybrid retainer combines elements of both a general retainer (enabling the
availability of the lawyer) and special retainer (providing for future services)
Illegal Fees
- Illegal fees are prohibited implicitly by M.R. 1.5(a) and 8.4
- Two most important types of illegal fees: fraudulent fees and violations of statutory
limits on fees
o Also include: fees collected by a public official when a statute prohibits the
private practice of law while in office; fees above those awarded by the court
in cases where the court has exclusive power to decide fees; and fees
collected for an illegal purpose
Fraudulent Billing and Fee Padding
- Fraudulent billing is a crime under both state and federal law
- Includes a wide range of conduct:
o Outright fraud such as charging for time that was not devoted to a clients
matter; double-billing for the same time; churning or overstaffing a matter
by billing a client for duplicative, excessive or unnecessary work; and charging
the client for firm overhead (e.g. duplicate costs, fax charges or secretarial
support)
Statutory Fee Limits
-

Various federal and state statutes limit the fees a lawyer can charge in particular
types of legal work

Fee Disputes
- Either a lawyer or client can bring a lawsuit to enforce or challenge a fee agreement
Fee Arbitration
- Lawyers and clients may agree to binding fee arbitration in their retainer agreement
o Clients are not required to arbitrate a fee dispute but a lawyer is required to if
the client requests it (M.R. 1.5)
Client Confidentiality in Fee Disputes
- A lawyer may reveal client confidences to collect a fee or establish a defense in a
dispute with the client over the fee, but only to the extent necessary to support the
lawyers claim or defense (M.R. 1.6(b)(5))
Contingent Fees
- In a contingent fee arrangement, the fee amount is contingent on the result the
lawyer obtains (i.e. personal injury case: lawyer receives a percentage of plaintiff
clients recovery)

65

Contingent fees are viewed as serving three valuable functions:


o Access or Financing Function: They enable persons who could not otherwise
afford counsel to assert their rights, paying their lawyers only if the assertion
succeeds (essentially, the client borrows from the lawyer against future
earnings to pay for the lawyers services)
o Incentive Function: Contingency fees give lawyers an incentive to seek their
clients success and to encourage only those clients with claims having a
substantial likelihood of succeeding
o Risk-Bearing or Insurance Function: Such fees enable a client to share the risk
of losing with a lawyer, who is usually better able to assess the risk and to
diversify by undertaking similar arrangements in other cases
Model Rule 1.5(c) specifically permits contingent fees unless some other law or
rule prohibits them
- Requires all contingency fee arrangements to be in writing and signed by the client
- Must include information on how expenses are to be allocated, and requires the
lawyer to provide the client with a written statement upon completion of the
representation
Cases on Excessive Contingent Fees
- Courts finding contingent fees to be excessive generally focus on one of four
problems:
o There was a high likelihood of substantial recovery by trial or settlement, so
that the lawyer bore little risk of nonpayment
o The clients recovery was likely to be so large that the lawyers fee would
clearly exceed the sum appropriate to pay for services performed and risks
assumed
o The percentage rate is excessive
o The base against which the percentage is applied is excessive or otherwise
unreasonable
Prohibitions on Use of Contingent Fees
- Divorce Cases
Model Rule 1.5(d)(1) prohibits any fee in a domestic relations matter, the payment or
amount of which is contingent upon the securing of a divorce or upon the amount of alimony
or support, or property settlement in lieu thereof
- Understand that there are some cases where you cannot take a contingency fee
(usually criminal or family/divorce law)
o Rationale for this rule is that it contravenes the strong public interest in
preserving marriage such a fee creates incentives inducing lawyers to
discourage reconciliation and encourages bitter and wounding court battles
o These days: it is likely for the lowered income spouse to require that the other
spouse pay their lawyer fees
- Criminal Defense Work
Model Rule 1.5(d)(2) prohibits contingent fees for representing a defendant in a criminal
case
o A contingent-fee agreement for criminal representation might affect the
clients choice in accepting a plea bargain or risking a trial; furthermore, they
might promote unscrupulous representation
Contingent Fees in Transactions
-

Some firms engaged in transactional work charge a fee contingent entirely or in part
on the transaction closing
Reverse contingency fee the contingency is the amount of money saved by the
client

66

Quantum Meruit Recoveries by Contingent Fee Lawyers who withdraw or Are Discharged
-

Old Rule: A discharge without cause was treated as a breach of the contingent fee
contract; therefore, the discharged lawyer could recover the agreed upon fee if the
client subsequently received an award
Modern Rule: recovery in quantum meruit should replace recovery on the contract in
case of discharge
o Under the rule of quantum meruit, the client is protected since the discharge
of an attorney is not always caused by a clients dissatisfaction with the
quality of service rendered but, rather, may result from the clients lack of
faith and trust or confidence in the attorney
o An attorney who substantially performs may be entitled to the full price of the
contract but it would be inequitable to force a client who has received no
service from the discharged attorney to pay the full price of the contract

Model Rule 1.8(i)(1) permits a lawyer to acquire a lien authorized by law to secure the
lawyers fee or expenses
Model Rule 1.15 will determine how fees are held (unearned vs earned monies)

Lawyer-Client Transactions
Transactions with Clients
Model Rule 1.8 covers a variety of lawyer-client transactions
- Business transactions 1.8(a)
- Media rights in the clients story 1.8(d)
- Gifts 1.8(c)
- Sexual relations 1.8(j)
Business Transactions
- M.R. 1.8(a) attempts to address the problems of lawyer overreaching and undue
favoring of her own interests at the expense of the clients interests
- Under Common Law, a lawyer in a transaction with a client is treated as a fiduciary.
As a fiduciary, the lawyer has the burden of proving that the transaction is fair and
equitable to the client. Remedies include rescission of a transaction and civil liability.
IMPUTATION RULE 1.10
-

No ethics rules that discipline a firm; only ethics rules that discipline individual
lawyers
- Sometimes this individual lawyers discipline can be traced to a firm
o If they wanted to disqualify the firm they could have but there is still the
idea of imputation
Committee On Professional Ethics and Conduct of Iowa State Bar Association v.
Mershon
stranger rule the lawyer who enters into a business transaction with a client
must give the client the same legal advice and guidance as would be given by a
lawyer who is a stranger to the transaction
Facts
Respondent, Iowa attorney Mershon, served as personal attorney to local farmer
Leonard Miller for 19 years. Miller wished to develop some of his farmland, but
would not be able to pay for Respondents services or the services of the
engineer he wished to help him develop it. In lieu of such fees, the three men

67

Issue
Holding

Reasoni
ng
Commen
ts

agreed to form a corporation in which their services would be provided in


exchange for shares in the property. The development deal fell through shortly
before Millers death, however, and Respondent (who was also executor for his
estate) was recommended to the board for discipline regarding his participation
in the transaction.
Was Respondents conduct here a violation of Iowas rule against going into
business with clients when each party has differing interests?
Yes. Respondent and Miller clearly had differing interests here, in that
Respondents fee was tied to the success of the corporation and he was
effectively contracting to make himself a debtor of the corporation to ensure
that his services would be performed. Miller also failed to make anything
approaching the full disclosure required by the relevant Iowa rule. Reprimand
recommended.
- Attorneys should generally avoid going into business with their
clients. Those willing to take this risk must recognize that courts
will always view the transaction in the light most favorable to
the client should the situation go awry.
- Value of legal fees was estimated with no real research done
- Nothing was done to assure that Miller would get farm back if either
Schenk or Mershon did not perform their duties
- Nothing was done to protect Miller or his estate in the event of the death
of any of the parties
- Mershon as a lawyer could have done three things differently:
- Refuse to participate personally in the transaction
- Recommended that Miller obtain independent advice
- Attempted to meet the high standard of disclosure
- Because he did not recommend Miller obtain independent advice, he was
obliged to make full disclosure but this was not done, therefore violation

Lawyer Equity Investments in Clients

Passante
Facts

Corporate lawyer gives company (Upperdeck) 100k loan for 3% of


shares
Some years later, Passante wanted repayment but the co refused
Jury awards him $33M
However, judge: if oral promise was bargained for and enforceable, it
was obtained in violation of Passantes ethical obligations as an
attorney
Court sets aside the payment for a violation of ethical obligations:
ABA Committee on Lawyer Business Ethics:
o Lawyers can satisfy the reasonable fee requirement by getting
paid in stock whose value at the time of payment equals
reasonable fees
o Lawyers must provide full disclosure of the risks of the
transaction, including the possibility that conflicts of
interest might develop that could necessitate the
lawyers withdrawal from the representation
o Should have told the co the need for independent
counsel (1.8(h))
o Law firms can minimize the risks of conflict by adopting policies
limiting the investment to an insubstantial percentage of stock

68

and the amount invested in any single client to a nonmaterial


sum and policies requiring that decisions regarding a firm
lawyers potential client conflict be made by someone other
than the lawyer with the principal client contact . . . or a partner
with no stock ownership in the client
Contracting with a Client for Rights to the Clients Story
- Prior to the end of the representation, a lawyer may not negotiate or enter into an
agreement, that gives the lawyer the literary or movie rights to a clients story, if
those rights are based in substantial part on information relating to the
representation M. R. 1.8(d)
- The prohibition ends with the representation
Gifts from Clients
- Gifts are distinguished from business transactions because they involve a transfer of
a thing of value without consideration
- M.R. 1.8(c) imposes significant limitations on lawyers receiving gifts from clients
client consent cannot overcome the prohibitions
Sexual Relationship with Client
- M.R. 1.8(j) prohibits a lawyer from having sexual relations with a client unless a
sexual relationship predated the lawyer-client relationship
Handling Property of Clients and Others
- The fiduciary obligations concerning property of others in a lawyers possession are
segregation, notification, record keeping, accounting and delivery these help to
reduce the danger of conversion, negligent misappropriation or loss and protect the
property from seizure by creditors of the lawyer or of other clients
- With respect to segregation, M.R. 1.15(a) requires a lawyer to hold property of clients
or third persons that is in a lawyers possession in connection with a representation
separate from the lawyers own property; funds must be kept in a separate account
from a lawyers own personal or business funds
- With respect to notification, M.R. 1.15(d) requires the lawyer, upon receiving funds or
other property in which the client or third person has an interest to promptly notify
the client or third person to enable them to protect their interests
- With respect to delivery, M.R. 1.15(d) requires the lawyer to promptly deliver to the
client or third person any funds or other property that the client or third person is
entitled to receive
- M.R. 1.15(e) requires the lawyer to keep disputed property separate until the dispute
is resolved
Commingling Funds
- This rule is violated whether the lawyer deposits the clients money in the lawyers
account or the lawyers money is deposited in the clients trust account
Record Keeping
- M.R. 1.15(a) specifies that a lawyer keep complete records for five years following the
termination of the representation
Client Protection Funds and Other Regulatory Safeguarding
- When lawyers fail to safeguard client property, they may be subject to discipline, a
civil suit for breach of fiduciary duty, or criminal sanctions for conversion
Keeping Money of Multiple Clients in One Account
- The rules allow a lawyer to use one trust account to hold the funds of multiple clients

69

The lawyer must maintain records that adequately identify the share of each person

Billable Hours
- This has become an important aspect of many law firms
- Examples of Non-Billable Activities
o reading advance sheets, newspaper, checking email, eating, bathroom breaks,
coffee, chatting with office neighbours, non-billable projects, training
sessions, CLEs, pro bono cases, organization of the office, submitting time
sheets, recruiting efforts
- Being able to bill accordingly is a part of being practice ready
Model Rule 1.5 requires it to be a reasonable fee
Model Rule 1.5(b) (c) and (e) are what you could have billed if you werent a lawyer
- Cannot split fees with nonlawyers as a general rule
- May pay a referral fee
Model Rule 1.8(f) third parties may pay your fee but you have to make sure that certain
circumstances exist
- Have to make sure professional independence is not jeopardized
- Client must say that it is okay (give consent)
- Must obtain clients consent confirmed in writing
Model Rule 4.1 if there is a third-party paying the fee, you may have a duty to make sure the
information you give is materially correct and that you dont admit anything under 4.1(b)
Model Rule 8.4
Scope of Lawyers Authority
- The client, with more at stake but in a position of vulnerability, has the right to decide
the objectives of representation
o Subject to Model Rule 1.2(c), which allows the lawyer, by agreement with the
client, to limit the scope of the representation if the limitation is reasonable
under the circumstances and the client gives informed consent
- The lawyer, with the greater technical knowledge and expertise, has the right to
choose the means by which they are to be pursued
o This is limited by the clients right to receive information as provided in Model
Rule 1.4 and the lawyers obligation under Model Rule 1.4(a)(2) to reasonably
consult with the client about the means by which the clients objectives are to
be accomplished
Things to KNOW:
1. A lawyers act or omission will bind a client when:
a. A lawyer has actual authority to make the decision Actual authority
can exist in:
i. Implied form (to take certain actions on behalf of the client without
asking the clients permission in advance, including matters of
professional courtesy and spot decisions made during trial)
ii. Express form
b. The client has lead a 3rd person to believe, reasonably, that the lawyer has
the necessary authority apparent authority
2. Generally speaking, the law governing lawyers vests the authority of making
decisions on the lawyer when the matters involve:
i. The means
ii. The procedures
iii. The tactics, and
iv. The strategies

70

3. When the decision involves objectives and goals of the representation, the
decision making authority vests in the client
4. By law, there are some decisions that are reserved only to the client.
a. EX: a guilty plea in a criminal case
b. EX: settlement in a civil case
Actual and Apparent Authority
- Actual Authority: May be either express, implied or inherent
o Actual authority exists when the principal (the client) through words
(express) or deeds (implied) causes the agent reasonably to believe that she
has the authority to act
o Implied authority can be based on the lawyers reasonable interpretation of
express instructions given by the client in light of circumstances faced by the
lawyer, or from the customary allocations of responsibility between lawyer
and client in particular situations
o Inherent authority flows from legal rules delegating authority on some
matter to the lawyer independently or client counsel
- Apparent Authority: Exists when the principal (i.e. the client) through words or deeds
causes a third party reasonably to believe that the agent has the principals authority
to act
o Courts generally require a showing of reliance and good faith on the part of
the third party
- Implied, apparent, and express authority need to be known Telemeter is a
good case for that
Rules to know:
o If a law vests a decision in a lawyer (see above), the lawyers implied
authority binds the client even in the face of a contrary view by the
client
o If the law vests decision making authority in the client, the lawyer
may bind the client if the lawyer has express authority or as to some
matters, the lawyer may also bind the client if the lawyer has
apparent authority
The exception to this is when it comes to settlements in a civil case
some circumstances where the court relies on apparent authority (but
more cases where they do not)
Model Rule 2.1 important consideration (while my representation of a client is not
necessarily a representation of a clients point of view, I am permitted to bring my own point
of view this of course, with the clients permission)
- Rendering advice to a client can take other things into account other than law itself
moral, economic, social and political factors that may be relevant to the clients
situation. Advocacy is not limited.
Model Rule 1.2(c) balance this rule with 1.8(h)
- Limitation of the scope if it is reasonable under the circumstances and the client
gives informed consent. The rule itself does not tell you how to (the limitations) or
the protections for the lawyer in those circumstances
Model Rule 1.4(a)(2) reasonably consult with the client on how to represent them (does not
say the lawyer is responsible for the means)
Attorney Judgment Rule
If a lawyer uses the exercise of judgment in pursuing a case (Whether it is the issue of what
expert am I going to call? What question am I going to ask that expert? How will I argue this

71

motion before the court? Etc.) there is a level of judgment a lawyers judgment will be
protected
- This is an issue of LAW not an issue of fact
Therefore, there is SOME decisions that lawyer can make; must keep client informed but can
make few decisions by themselves!
Cannot sue your lawyer for this if they made a judgement that was reserved for the lawyer
(they had authority) such as questions to ask witness, motions to bring pre-trial, how to
structure opening and closing statementssince these have to do with the lawyers training
they have the best knowledge of what is allowed.
Smith v. Lewiswhile judgments are protected, you need to make a judgement and cannot
rely on things being fortuitous without knowing anythingignorance of the law.
Model Rule 1.2 Means of achieving the objectives are usually reserved to the lawyer, but the
objectives of the representation are reserved for the client.
Keep client reasonably informed, consult with the client 1.2(d).
Allocation of Decision-Making Authority in Civil Litigation
International Telemeter Corp. v. Teleprompter Corp.
Facts
Case arose out of a patent infringement suit that ITC had brought against
Teleprompter. Principals of the two companies had participated in extended
negotiations and agreed on basic terms of settlement. Bresnan, Teleprompters
president, had signed off on the final documents but when Teleprompters
management changed, the new management refused to deliver the settlement
documents. Kirsch contacted the other lawyer and tried to withdraw from the
settlement deal
Issue
Whether there was sufficient evidence to support the trial courts conclusion that
Kirsch, Teleprompters lawyer, had either actual or apparent authority to bind
Teleprompter to a settlement
Holding
The decision made by the trial court forcing the defendant to honor the
agreement was correct because the defendants attorney had acted within his
authority and the plaintiffs reliance on the same was right as long as it did not
believe he was ever exceeding his authority.
Ratio
- In this case, the judge upholds the settlement based on the lawyers
apparent authority to bind Telepromptor
o Note the reference to actual authority as well
Reasoni
o This is not just a break down between attorney and client
o This is a break down for the client itself
ng
o Lawyers work for a purpose - The court will not hold that the entity no longer
holds liability bc of a change of management
o There is a point at which the clients will will push the lawyer too far
The lawyer then needs to be concerned with zealous advocacy
o Rule 1.2 scope of representation
Look at this rule when questioning whether a lawyer and/or client had
the authority to do something
Commen
- 2 different opinions that agree on the result but disagree as to how it is
you get to that result
ts
o Majority believed it was implied by the status of the case at the

72

time it was filed (Implied Authority: it is understood as under a


lawyers authority to carry out the representation)
o The concurring opinion believed there had been an express
authorization at the time of acceptance (informed of the
authority given to the lawyer)
Model Rule 1.2(a) allegation of authority between client and lawyer

Ending the Relationship

Difference between litigation and non litigation.


Lawyer Withdrawal
Mandatory Withdrawal
Model Rule 1.16(a) deals with mandatory withdrawal, as well as mandatory
refusals to accept representation
- Withdrawal is mandatory when the continuing representation will result in a violation
of ethics rules or other law
- Another common reason for withdrawal is conflict of interest, as well as to avoid
violating other ethics rules (i.e. duties of competence and diligence), or when the
lawyer is too ill to continue the representation
Permissive Withdrawal
Model Rule 1.16(b) states the circumstances under which a lawyer may withdraw
from representation, in effect, fire the client
- If the matter is before a tribunal, the court must grant permission to withdraw (and
may require a lawyer to undertake the representation in the first instance)
- A lawyer may withdraw for any reason or no reason as long as withdrawal can be
accomplished without material adverse effect on the client
- All 7 conditions are freestanding; you need to only meet one. The OR
- Catch all, 7th condition
- The hot potato doctrine limits the lawyers ability to resolve a conflict of interest
between a new and a current client under M.R. 1.7 by dropping the current client like
a hot potato and continuing with the representation of new client under
circumstances permitted by M.R. 1.9. The doctrine requires withdrawal from both
representations
Protecting a Client upon Withdrawal
- Whether a lawyer is required or permitted to withdraw, and regardless of the
circumstances surrounding withdrawal, withdrawal must be accomplished in
accordance with M.R. 1.16(d), which requires the lawyer to take steps to protect the
clients interests
Discharge and Its Consequences
- In general, a client may discharge a lawyer at any time with or without good cause.
The client cannot waive this right; any attempt to contract away the right to
discharge the lawyer is likely to be held unenforceable as against public policy
o Approval of the tribunal is required under M.R. 1.16(c) a court is likely not
to approve when a client attempts to change lawyers on the eve of or during
trial
When the request is made after the trial has begun, the defendant
must show good cause, such as an actual conflict of interest, a

73

o
o
o

complete break-down in communication or an irreconcilable conflict


with his attorney
The client may feel practically constrained to keep a lawyer despite the right
of discharge (such as when a public agency or subsidy provides counsel to a
poor person)
The clients ability to discharge a lawyer at will does not mean that discharge
has no legal and financial consequences
The contract between the lawyer and client, or restitutionary principles, may
allow a lawyer to recover a portion of the lawyers fee regardless of the reason
for discharge

Ending the Relationship Before it Starts


Togstad v. Vesely, Otto, Miller & Keefe
Facts
- Plaintiff, Joan Togstad, is brought suit against Defendant, attorney Jerre Miller,
for his alleged negligence in the course of a preliminary consultation with
her. After she had fully described the conditions leading to a serious accident
suffered by her husband while undergoing treatment in a local hospital,
Defendant told her that it was his opinion that she probably didnt have a
case, but that he would consult with his partner and get back to her.
- Since he never followed up with her, she assumed that his initial assessment
was correct and did not bring suit in reliance on this opinion. She did not
consult with another attorney until one year after her discussion with
Defendant, at which time the statute had run and her potential claim was
barred. Plaintiff was awarded nearly $650,000, and Defendant now
challenges this award.
Issue
Did Miller commit malpractice under these facts?
Holding
- Yes. An attorney-client relationship was formed under these facts, and
Defendant acted negligently in assessing the merits of Plaintiffs case.
o He had reason to know that his advice could injure Plaintiff if
negligently given, and had a duty to further investigate the case
before indicating that she did not having a valid claim.
- Note that an attorney-client relationship was immediately created when
Plaintiff entered Defendants office seeking his opinion, despite the fact
that she never formally retained him to represent her.
- **He must have told her about the statute of limitations if nothing else
standard of care
o And that he was not her lawyer
Ratio
- An attorney-client relationship was immediately created when
Plaintiff entered Defendants office seeking his opinion, despite the
fact that she never formally retained him to represent her.
- Had this been an ethics case, he would have violated 1.4
(communications)
- It is a civil case lawyer should have told her about the statute of limitations
- This is a very influential case
Reasoni
- In a legal malpractice action of the type involved here, four
elements must be shown:
ng
(1) that an attorney-client relationship existed;
(2) that defendant acted negligently or in breach of contract;
(3) that such acts were the proximate cause of the plaintiffs damages;
(4) that but for defendants conduct the plaintiffs would have been
successful in the prosecution of their medical malpractice claim
Commen
Miller never thought he was her attorney, she did and her belief was

74

ts

reasonable
Lewis v. Smith Ignorance is not an excuse. Negligent
If Miller didnt want to take her case, he should have written a letter.
Anything which is unambiguous
Court says, he should have told her about the Statue of limitations

75

CHAPTER 2 CONFORMITY TO THE LAW

Pages 57 76; 93 118;

Criminal Law - Crimes Related to False Testimony


United States v. Gellene
- See above! (Perjury)
Tort Law Claims by Non-Clients
Negligent Misrepresentation
Greycas, Inc. v. Proud
Facts
o Defendant, attorney Theodore Proud, wrote a letter to Plaintiff, Greycas, Inc.,
stating that his brother-in-law Crawfords farm equipment had no prior liens
on it to assist him in securing an emergency loan
o After Crawfords suicide, Defendant learned that the equipment was already
heavily indebted and that Plaintiff had never performed the tax and
judgment searches that he had claimed to have completed in his letter
o Plaintiff is claiming negligent misrepresentation. Defendant argues that he
had no duty of care to the Plaintiff in this situation.
Issue
May an attorney be liable for negligent misrepresentation to an adversarial
party?
Holding
Yes. Proud may be liable here as someone who generally supplies information
and opinions to others in the course of their business transactions. Judgment
stands, and Defendant is recommended for discipline.
o Proud wrote this letter knowing that Greycus was going to rely on it
Reasonin - It is not entirely clear why Plaintiff is claiming negligent malpractice rather
than outright fraud, which would have undoubtedly been more
g
straightforward. Defendant clearly lied in his statement and claimed that tax
and judgment searches had been performed when they had not therefore,
clearly ulent actions.
- For a non-client to succeed in a negligence action against an
attorney, he must prove that the primary purpose and intent of the
attorney-client relationship itself was to benefit or influence the
third party Pelham
- Proud, in the practice of his profession, supplied information (or rather
misinformation) to Greycas that was intended to guide Greycas in
commercial dealings with Crawford. Proud therefore had a duty to use due
care to see that the information was correct. He used no care.
Commen - 1.7(a)(2)
- Conflict of interest, as it was the brother in law and so blood relationhow
ts
would he be able to be unbiased in representing Greycas
- If you had not searched (when it is within your responsibility) you might be in
trouble in terms of Model Rules 4.1(b) [Truthfulness in statements to
others] where you are not allowed to fail to disclose a material fact when it is
necessary to avoid assisting in a criminal or fraudulent act by a clientonly
exception is if the material fact is covered by 1.6.
o Since the representation was taken on by Proud, there was nothing
prohibiting such a claim to move forward for disciplinary action

76

The lawyers duty under criminal and civil law to refrain from assisting a client
in conduct that is illegal. A lawyer violates that duty if:
1. The client is engaged in a course of conduct that violates the criminal law or is an
intentional violation of a civil obligation, other than failure to perform a contract or
failure to sustain a good faith claim to property
2. The lawyer has knowledge of the facts sufficient to reasonably discern that the
clients course of conduct is such a violation; and
3. The lawyer facilitates the clients course of conduct either by giving advice that
encourages the client to pursue the conduct or indicates how to reduce the risks of
detection, or by performing an act that substantially furthers the course of conduct
Professional Malpractice
- Standard of care determined by the skill, knowledge, prudence, and diligence,
brought together on similar matters by a lawyer of ordinary competence
Tort of Malpractice is made up of the following elements:
1. A duty (usually established by attorney-client privilege)
2. A breach of a duty
3. Causation
4. Harm
Intentional Torts
- Generally, the law grants lawyers no privilege to commit (or assist clients in
committing) intentional torts against third parties.
- Intentional tort that poses the greatest threat to lawyers is fraud because a large part
of what lawyers do is communicate with others on behalf of their clients with the aim
of getting those others to give up their property or money in reliance on what the
lawyer says
Fraud and the Recklessness Standard of Intent
- The basic elements of the intentional tort of fraud are:
o (1) a material false statement,
o (2) made with an intent to deceive,
o (3) which is reasonably relied on by a person to whom it is made
o (4) to that persons detriment
- Fraudulent intent may be shown by a reckless disregard for the truth or falsity of the
proposition asserted
Breach of Fiduciary Duty
- Lawyers are fiduciaries for their clients and thus may be liable for this tort or sued for
constructive fraud (i.e. for failing to disclose something important or material to the
beneficiary or client)
Exceptions and Qualifications
- Lawyers representing clients in litigation have a virtually absolute privilege to make
defamatory statements about non-clients in court (whether those statements are oral
or in writing) or in settings that are reasonably related to pending or contemplated
litigation
o As well as fiduciaries, a lawyer has a qualified privilege to make statements
intended to protect the interests of others

77

Although lawyers are not immune from suits for malicious prosecution or abuse of
process, these torts are notoriously difficult to plead and prosecute successfully
Lawyers are generally not liable for intentional interference with contractual relations
simply for advising clients to breach a contract

Third-Party Legal Opinions and Ethical Guidance


-

Parties to business transactions often seek legal reassurance on certain issues from
opposing parties counsel as part of due diligence in completing a transaction
known as third-party legal opinion
o Typically takes the form of a formal letters (sometimes called a comfort letter)
- Opinions often deal with such matters as legal authority to engage in a transaction,
valid incorporation, the enforceability of the transaction contracts, and compliance
with particular laws, such as securities laws
Model Rule 2.3 set forth the lawyers ethical responsibilities in writing opinion letters
Assisting a Client in Tortious or Illegal Conduct
- Lawyers can be held liable not only for primary violations but also for secondary
violations, usually referred to as aiding and abetting or conspiracy
Hazard: How Far May a Layer Go in Assisting a Client in Unlawful Conduct?
- How far may a lawyer lawfully go in providing assistance to a client that might enable
the client to carry out an act that is to some degree illegal?
- Spectrum where at one end the lawyer can provide advice (without specific aid or
encouragement to the client) and on the other end is pure instrumentalism a
lawyers physical execution of a purpose that the client would like to realize but
cannot or will not actually execute himself (i.e. lawyer who serves as bagman in an
illegal payoff)
o Law clearly permits conduct at one end of the spectrum and prohibits conduct
at the other
- The dimensions of the lawyers duty under criminal and civil law to refrain from
assisting a client in conduct that is illegal. A lawyer violates that duty if:
o (1) The client is engaged in a course of conduct that violates the criminal law
or is an intentional violation of a civil obligation, other than failure to perform
a contract or failure to sustain a good faith claim to property;
o (2) The lawyer has knowledge of the facts sufficient to reasonably discern that
the clients course of conduct is such a violation; and
o (3) The lawyer facilitates the clients course of conduct either by giving advice
that encourages the client to pursue the conduct or indicates how to reduce
the risks of detection, or by performing an act that substantially furthers the
course of conduct
Model Rule 3.4(e) mandates that a lawyer shall not . . . allude to any matter that the
lawyer does not reasonably believe is relevant or that will not be supported by
substantial evidence . . .
Model Rule 3.4(e) prohibits a lawyer from asserting personal knowledge of facts in
issue except when testifying as a witness, or stating a personal opinion as to the
justness of a cause, the credibility of a witness, the culpability of a civil litigant or the
guilt or innocence of an accused
Model Rule 3.6 Test for a Lawyers Pre-Trial Speech (Publicity)
Lawyers are held to a different standard, they do not have the intent and are subject
to the objective standard unlike general citizens

78

Chapter 7: Litigation (Part II)


Frivolity, Harassment and Delay: Professional Rules and Attitudes
- The fundamental purpose of the adversarial justice system is the fair accurate and
efficient resolution of disputes and so to protect such a goal lawyers and litigants
must be deterred to the extent practicable in an imperfect world from conduct that
will endanger the goals of the system by producing unfair and inaccurate results and
needless expense.
- Prohibiting the use of the court system to present frivolous claims is a reasonable
restriction that does not deprive the protestor of the many other outlets for
expression of grievances (a time, place, and manner limitation on First Amendment
rights of freedom of speech)
Model Rule 3.1 states that a lawyer (other than one for a criminal defendant or a person
subject to incarceration) shall not bring or defend a proceeding, or assert or controvert an
issue therein, unless there is a basis for doing so that is not frivolous, which includes a good
faith argument for an extension, modification or reversal of existing law
- Cmt [2] this does not include a duty to make a full inquiry about the underlying facts
before acting but they are to inform themselves about the facts of their clients cases
and applicable law and determine if a good faith argument can be made to support
the clients position
- It is tough to prove there are violations of this rule
Model Rule 4.4(a) states only that a lawyer shall not use means that have no substantial
purpose other than to embarrass, delay or burden a third person (leads to few disciplinary
actions, and deals with colorable claims)
- A frivolous position is one that a lawyer of ordinary competence would recognize as
so lacking in merit that there is no substantial possibility that the tribunal would
accept it
- RULE 11 permits lawyers to make arguments for the extension, modification, or
reversal of existing law or for the establishment of new law as long as they are
nonfrivolous
Model Rule 3.2 imposes on lawyers the affirmative obligation to make reasonable efforts to
expedite litigation but then adds consistent with the interests of the client
- Cmt [1] test
o Whether a lawyer acting in good faith would regard the course of action as having
some substantial purpose other than delay. Realizing financial or other benefit
from otherwise improper delay in litigation is not a legitimate interest of the client
Rule 11 Sanctions
Representations to Court. By presenting to the court a pleading, written motion, or
other paper whether by signing, filing, submitting, or later advocating it an
attorney or unrepresented party certifies to the best of the persons knowledge,
information, and belief, formed after an inquiry reasonable under the circumstances:
(1) It is not being presented for any improper purpose, such as to harass, cause
unnecessary delay, or needlessly increase the cost of litigation;
(2) The claims, defenses, and other legal contentions are warranted by existing law
or by a nonfrivolous argument for extending, modifying, or reversing of existing
law or for establishing new law;
(3) The factual contentions have evidentiary support or, if specifically so identified,
will likely have evidentiary support after a reasonable opportunity for further
investigation or discovery; and
(4) The denials of factual contentions are warranted on the evidence or, if specifically
so identified, are reasonably based on belief or a lack of information

79

Rule 11 is the federal rule regarding civility


1983 Rule11 was stiffened and required judges to sanction lawyers whenever a paper
filed in court by a lawyer was not well grounded in fact or not warranted by existing
law or a good faith argument that existing law should be modified or reversed
o This was an objective standard
o Also required to conduct a reasonable investigation into the facts and law
before filing papers in court, or sanctions would result
December 1993, a new version came into effect of Rule 11 (Amended)
Effects:
o To expand/strengthen the rigor of some aspects of the rule
o To soften other aspects of the rule
o Expanded somewhat the lawyers duty of candor
o Sanctions for violations were no longer mandatory and a safe harbor provision
was added that immunizes violations if the violating lawyer withdraws or
corrects the offending statements within 21 days of her opponents calling
them to her attention
o Some argued this rendered the rule toothless
December 2007slightly reworded for clarity and uniformity
Should know that the 1993 amendments expanded the scope and the
strengthened the rigor in two ways:
(1) it imposed a continuing obligation to review past actions that may have
been justified from the paper (pleadings/motions) was filed but are no
longer justified (the old rule only applied to the filed paper): and
(2) it made it clear that a law firm is responsible when one of its lawyers signs
a paper (the old rule was limited to the person who signed it)
Three other amendments softened the rule
(1) it created a safe harbor provision giving 21 days after notification for the
individual to withdraw
(2) it allowed for the imposition of sanctions to be made discretionary with
the trial court rather than mandatory
(3) the award of attorneys fees to the moving party is limited to improper
purpose cases not just a bad filing (the old rule gave not only mandatory
sanctions but also required attorney fees to be paid by the losing side to
the winning side now this is discretionary)
2007 version
o Protects a plaintiffs lawyer who, in making a factual allegation, notes that the
allegation is likely to have evidentiary support after a reasonable opportunity
for further investigation
o Protects a defense lawyer when in denying a factual contention, they note
that the denial is reasonably based on a lack of information and belief
o But both only to the extent it remains reasonable to have failed to discover
the fact of the matter
o Not applicable to discovery motions or other papers that are filed in
connection with discovery
Rules to be aware of when it talks about frivolous proceedings
o M.R. 3.1

Affirmative Duty to Investigate


- Rests with the lawyer who files a complaint or answer, makes a motion, or submits
any other paper to the court

80

The reasonableness of the investigation is determined by factors such as the time


available for the investigation, the costs to the adversary, the experience of the
attorney, and other relevant circumstances
Reliance on facts provided by the clients to establish a prima facie case is often
sufficient, but use caution
Use of discovery is permitted to bring out facts in the hands of the opposition
A Rule 11 motion must be served separately from other motions and only after the
targeted party has been given the opportunity to withdraw the challenged statement
within 21 days of service of notice
Must be through proceeding cannot be after final judgment, and now sanctions are
discretionary

Other Procedural Sanctions


Unreasonably and Vexatiously Multiplying Proceedings
Inherent Judicial Authority to Sanction Bad Faith Conduct
- Inherent authority in federal court to impose sanctions, including an award of attorney
fees when the opposing party has acted in bad faith, vexatiously, wantonly, or for
oppressive reasons. (Roadway Express Inc. v. Piper upheld this authority (SC case))
- Chambers v. NASCO, Inc.the Supreme Court held that the federal courts have inherent
power to sanction even when sanctions would be unavailable under Rule 11 or s. 1927,
and this allows judges to sanction lawyers, as well as non-lawyers, when the conduct
that interferes with the administration of justice falls outside the four corners of other
rules or statutes that authorize sanctions.
Sanctions for Discovery Abuse
Qualcomm v. Broadcom
Facts
Lawyers for a technology company in a patent dispute allegedly withheld
thousands of pages of documents that would have contradicted the position
taken by their client in litigation.
If the company was working with JVT they would have a duty under patent law to
disclose technology allegedly covered by the patent at issue.
The outside law firm, and the client repeatedly refused their involvement in
deposition and discovery requests. Qualcomm later admitted existence of emails
showing participation in JVT during trial, the lawyers attempted to claim they had
performed a reasonable search but this was denied.
Six outside lawyers, out of nineteen, were sanctioned for discovery abuse.
Issue
Whether Qualcomm had ever participated in a group called Joint Video Team
which was working on a standard for compressing video filed for efficient
transmission.
Rational Lawyers have an affirmative duty to conduct discovery in a responsible manner,
e
which includes conduct[ing] a reasonable inquiry to determine whether
discovery responses were sufficient and proper
Comme
Hearing for the sanctions was scheduled for 2010.
nts
Sanctions at the Appellate Level
-

Most often levied under Fed.R.App.P. 38.

81

If they determine an appeal is frivolous, it may after a separately filed motion or


notice from the court and reasonable opportunity to respond award just damages
and single or double costs.
Not mandatory sanctions here
Some have objective standard, some have subjective bad faith

The Anders Brief


- In Anders v. California, the court ordered that Counsel must accompany her request to
withdraw with a brief setting forth anything in the record that might arguably support
the appeal
- Given the Anders doctrine lawyers taking an appeal on behalf of criminal defendants run
little risk of Rule 38 sanctions
- Rule 3.4: fairness to opposing counsel

82

CHAPTER 9 COMPETENCE
Checks on Incompetence
-

Preventative measures to ensure that clients receive competent legal services begin
with bar admission requirements attendance at law school and passing the bar
exam

Ethics Rules on Competence


Model Rule 1.1
- A lawyer shall provide competent representation to a client
- In many instances, the required proficiency is that of a general practitioner, but in
some circumstances, expertise in a particular field of law may be required
Model Rule 1.3
- A lawyer must act with reasonable diligence, which means they must pursue a
matter on behalf of a client despite opposition, obstruction or personal inconvenience
to the lawyer, and take whatever lawful and ethical measures are required to
vindicate a clients cause or endeavor
Model Rule 1.4 identifies 5 specific aspects of the duty of communication
- Duty to communicate about decisions that require the clients informed consent
- Duty to consult about means to accomplish the clients objectives
- Duty to keep the client reasonably informed on the status of the matter
- Duty to comply with reasonable requests for information
- Duty to consult with the client about limitations on the lawyers conduct
Continuing Legal Education
-

Has been made mandatory in most states but there is no quality control of the
content of courses
Peer Review
-

Theory behind this is that lawyers may maintain and improve their competence by
submitting their practice methods to the scrutiny of knowledgeable colleagues for
comment and constructive criticism
Reputation and the Market
-

Reputation is established partly through the opinion of clients, but at least equally
through the opinion of other lawyers

Malpractice
- Theoretically available to any client who is harmed by a lawyers departure from
standards of ordinary care but usually not fought unless harm is significant and
there is a good chance of winning
Overview of the Tort of Malpractice
EXAM: What are the elements to the tort of malpractice?
- Duty
o A duty of care that is arising from the attorney-client relationship, or, in those
jurisdictions with exceptions to privity of contract, some other showing that a
duty is owed to the plaintiff
- Breach of duty
o A failure by the lawyer to exercise the care that reasonably competent lawyers
exercise under similar circumstances (knowledge, skill, prudence and
diligence)

83

Causation
o Have to show the conduct of the lawyer actually caused the harm in fact
(have to retry the initial case, which could have been won)
Actual and proximate cause of the plaintiffs injury
o Creates a circumstance of a case within a case
Harm
o Usually purely economic harm and not emotional/physical harm (would have
achieved a different and more advantageous result in the transaction or
litigation but for the lawyers conduct)
o Legally cognizable harm must have been cause by the lawyers act or omission

Model Rule 1.1 defines competence with a list:


- legal knowledge, skill, thoroughness and preparation reasonably necessary for the
representation
Standard of Care and Causation
Lucas v. Hamm
- The lawyer did not understand the rules against perpetuities and botched the estate
plan in question, the Court ruled that they did not understand RAP either as it was too
hard a concept to expect lawyers to know. The lawyer was not found liable for this
mistake
Smith v. Lewis
Facts
Plaintiff, Rosemary Smith, retained Defendant, attorney Jerome Lewis on Feb
1967, to assist her in obtaining a divorce in California. Using personal knowledge
rather than consulting the relevant law, Defendant advised her that her
husbands National Guard retirement benefits would not be considered
community property in California, and did not plead them as such in the
complaint filed three days later. Plaintiff was granted $400 a month in alimony
and child support from this claim on Feb 1968.
Plaintiff was later advised that she should have included these benefits as
community property (on July 1968), but this claim was by then time-barred.
Plaintiff proceeded to bring a malpractice claim against Defendant and received
$100,000 in damages after a jury trial, which Defendant now appeals.
- Defendant claims that law with regard to characterization of retirement
benefits was unclear as a matter of law an attorney is not liable for
mistaken advice when well informed lawyers in the community entertain
reasonable doubt as to the proper resolution of the particular legal
question involved
Issue
To what extent should Defendant be held liable for possibly misinterpreting an
ambiguous legal standard-using only his immediate knowledge of it-upon which
reasonable attorneys in the field could disagree? Should the entire award be
allowed?
Holding
Defendant is liable. Defendant should have consulted the relevant law before
advising Plaintiff, and was therefore properly subject to a malpractice claim.
Verdict and award upheld.
Ratio
General Rule: The standard is competence, not incompetence (Lucas v
Hamm is the sole exception of this rule)
Reasoni
- Defendant failed to conduct any reasonable research
- The defendants refusal to educate himself to the applicable principles of
ng
law constituted negligence which prevented him from exercising

84

Commen
ts

informed discretion with regard to his clients rights


Even as to doubtful matters, an attorney is expected to perform
sufficient research to enable him to make an informed and intelligent
judgment on behalf of his client.
There is nothing strategic or tactical about ignorance
o Lawyer cannot have the benefit of not being liable when you have
not done the required standard of work, and tried
Lucas v. Hamm is not available
Standard of care is determined by the skill, knowledge, prudence
and diligence brought to bear on similar matters by a lawyer of
ordinary confidence

Standard of Care
- Lawyer must know the principles of law that are either commonly known to attorneys
or which may be discovered through standard research techniques
Professional Custom Sets the Standard
- Professional malpractice differs from ordinary negligence in that the standard of care
is determined by the skill, knowledge, prudence and diligence normally exercised by
lawyers in similar circumstances
Expert Testimony
- Plaintiff in a malpractice action generally must produce expert testimony to establish
both the level of care owed by the attorney under the circumstances and the failure
to conform to that level of care
Causation of Harm
- If a lawyers breach of duty to a client occurs in the handling of litigation and the
harm suffered is the loss of a recoverable claim, the client will usually be required to
prove that the underlying case would have succeeded if it had been property brought
or litigated
- Proving causation is even more difficult in malpractice cases arising out of
transactional representation:
o Former client alleging malpractice in a transactional matter must prove that
the client would have gotten a better deal but for the attorneys errors
Other Issues
Violation of Ethical Rules as a Basis for Malpractice
-

Violation of an ethics rule does not create a civil cause of action or constitute
negligence per se. but the courts view ethics rules as relevant and admissible
evidence when relevant to the standard of care

Limiting Malpractice Liability by Agreement


Model Rule 1.8(h)(1): provides that a lawyer may prospectively limit her malpractice liability
to the client, but only if the client is independently represented in making the agreement
Model Rule 1.8(h)(2): prohibits a lawyer from settling a claim or potential claim for such
liability with an unrepresented client or former client unless that person is advised in writing
of the desirability
Malpractice Insurance
-

Malpractice insurance comes in two major forms:

85

o
o

(1) Occurrence insurance covers the lawyer for acts or omissions during the
policy term, regardless of when the claim is asserted; and
(2) Claims made insurance covers only claims made during the policy term,
regardless of when the act or omission took place

Role of Malpractice Insurers in Loss Prevention


-

Legal malpractice insurers may be able to provide loss prevention services more
cheaply and effectively than individual lawyers or law firms

Effective Assistance of Counsel Under the Sixth Amendment


- Sixth Amendment provides that in all criminal prosecutions, the accused shall enjoy
the right . . . to have the Assistance of Counsel for his defense
o Counsel must be appointed in every criminal case, including misdemeanors, in
which a prison sentence may be imposed, unless the defendant exercises the
right of self-representation
Ineffective Assistance: The Constitutional Standard
Strickland v. Washington
Facts
Respondent, Strickland, during a ten-day period, committed three groups of
crimes, including three brutal capital murders, torture, kidnapping and
attempted murders. Respondent pled guilty to all crimes and, stated that he
accepted responsibility for the crimes and had only acted under extreme mental
stress resulting from his inability to care for his family (pled guilty after his two
accomplices were caught).
Against the advice of his counsel, he waived his right to an adversarial jury at his
sentencing hearing, and chose to be sentenced by the Judge. Because the
defendant had already claimed extreme mental stress, his counsel used that,
along with information from his wife and mother to establish his character. He
did not request psychiatric evaluation, did not look for further mitigating
evidence, and did not request a pre-sentencing report he decided not to look
further for evidence concerning respondents character and emotional state.
Counsels decisions were said to reflect his hopelessness after his client pled to
all offenses. Strickland was sentenced to death, and he sought habeas corpus
relief due the failures of his counsel to come up with mitigating evidence.
Issue
Whether, after a defendant has pled guilty in a capital murder case, counsel has
a duty to present mitigating evidence, in order to meet the Sixth Amendment
standard for effectiveness.
Holding
The Supreme Court, in an opinion authored by Justice OConnor, held that while
counsel may have committed error, it was not so ineffective as to overturn a
death sentence.
Ratio
An error by counsel, even if professionally unreasonable, does not
warrant setting aside the judgment of a criminal proceeding if the error
had no effect on the judgment
Reasoni
Counsels arguments:
ng
- Respondents remorse and acceptance of responsibility justified sparing
him from death penalty
- Respondent had no history of criminal activity and he was under extreme
mental and emotional distress
Trial Judge:

86

All three murders were especially heinous, atrocious and cruel and
involved repeated stabbings
- All involved robbery, so were for pecuniary gain
- All murders were committed to avoid arrest for the accompanying crimes
Although respondent tried to claim counsel had rendered ineffective assistance
at the sentencing proceeding, the court found that the aggravating
circumstances proved to be completely overwhelming, and there was no chance
that the outcome would have been any different.
Court of appeal: punishment that a defendant faces is merely one of
the circumstances to be considered in determining whether counsel
was reasonably effective
STRICKLAND 2 PRONG TEST:
- Found the main question to be: the benchmark for judging any
claim of ineffectiveness must be whether counsels conduct so
undermined the proper functioning of the adversarial process
that the trial cannot be relied on as having produced a just
result.
-

Defendant must meet a two prong test in order to show that


assistance of counsel was so defective as to require reversal or
setting aside of a death sentence:
(1) the counsels performance must be deficient (requires
showing that counsel made errors so serious that counsel was not
functioning as the counsel guaranteed the defendant by the Sixth
Amendment), and
(2) that deficient performance must have prejudiced the
defendant so much as to have deprived him of a the right
to a fair trial; **See below for prejudice explaination

When judging the performance of an attorney, counsel must be given a


great deal of latitude, considering all circumstances. each case must be
considered on a case-by-case basis; and
Also, in a federal habeas challenge to a state criminal judgment, a state
court conclusion that counsel rendered effective assistance is not a
finding of fact binding on the federal court, but it is a mixed question of
law and fact.
Reasonable probability is a probability sufficient to undermine
confidence in the outcome

Commen
ts

dont know what is going to be produced until you submit something


ignorance cannot be bliss
real question whether or not there is negligence to not have had accused submitted for
psychological testing
o I dont know what its going to show is not an excuse not to do it
o have to find out what there is

If you can prove conflict of interest, you only need to prove the first
prong, not the second
**UNDERSTAND THIS YOU WILL SEE SOMETHING ON THE EXAM RELATED TO
STRICKLAND and ALSO ABOUT MALPRACTICE ABOVE**

87

It is hard to prove malpractice in criminal matters, as it is tough to win since


exoneration is needed to prove it (if malpractice claim can occur if there is a
Strickland violation)

Requires Showing of Prejudice


- Except in certain limited circumstances, the convicted defendant must show a
reasonable probability that, but for counsels unprofessional errors, the result would
have been different
Constitutional Rule of Defense Counsel
- According to Strickland, the purpose of the Sixth Amendment guarantee of effective
assistance of counsel is to ensure that the adversarial testing process works to
produce a just result, a fair trial whose result is reliable
- According to William Genego: the attorneys role is to do everything ethically proper
to see that the client receives the most favorable outcome possiblewhether or not
it produces an outcome which society considers just
When Prejudice is Presumed
- In Strickland the Court stated that prejudice will be presumed in certain limited
circumstances:
o When there has been an actual or constructive denial of the assistance of
counsel altogether
o Or when the government interferes with counsels assistance in such a way
that prejudice is so likely that case-by-case inquiry into prejudice is not worth
the cost
Malpractice Claims Against Criminal Defense Lawyers
- As a practical matter, criminal defense lawyers are not accountable to their clients in
malpractice actions
o First, many states require the convicted criminal defendant to be successful in
having the conviction either set aside by direct appeal or by collateral attack
before instituting a malpractice claim
o Second, an increasing number of jurisdictions require that the convicted client
prove that he was actually innocent of the offense charged before he can
succeed in a malpractice case
In re AMB, 248 Mich App 144 (2002)
Facts
- father of mentally challenged child is in jail
- child is transferred to hospital
- filed for court to terminate the fathers and mothers rights
- FIA filed for order that would allow the baby to diedenied
- FIA files for a different order in front of a different refereeconsiders the
same petition and ask who the babys lawyer is
o Referee finds none so appoints house counselemergency
counsel
o Baby dies on this day
Issue
The FIA and the Court was interested in this position because:
- Only time a ward of the state was killed while the state stood by and
watched without taking action to protect
- Wanted to become a ward of the child to take away the feeding of the
child (why would they do this whos interest is being served here,
because clearly the clients interest is not being served)
Ratio
An attorney must thoroughly prepare oneself for a any given client,
including adequate research and preparation for the legal matter at

88

Holding

Commen
ts

hand
Wanted to kill this baby because of the influence of a constituent of the abuse
and neglect court, which is the hospital
- Wanted some relief of its burden in relation to these kids
- The lawyer never had a clue that this was even going on
- They wanted to become the guardian of the child so that they could take
away the feeding and care of the baby to reduce their burden (hospital)
-

Rule 1.1 competence is tied to how much time you have to get ready
Baby was moved from Oakwood to DMConly neonatal high risk facility
in the areaDMC wanted the order because it cost a lot of money from
folks they never collect money from so they went to get an order to not
have to take care of the baby
DMC withdrew all feeding before the order was even ordered

CHAPTER 10 LAW, LAWYERS AND JUSTICE


Shapero, Ohralik and Primusset out a really good line that advertising is protected speech
for lawyers and others. Yes there are limits, for example it has to be truthful in its entirety.
State can put other limitations, to discourage the permitted speech (certain size to be used).
The in-person prohibition, gets first an exceptionthe purpose of in-person is for political
advantage or a cause (still permitted) not all solicitations are banned, for example if there is
a prior professional or political relationship with potential client this is allowed. The ones that
are not allowed dont always make sensefor example every other business can solicit, but
lawyers cannot. The anti-solicitation works more to protect established relationships then it
does to protect against new ones.
Advertising and Solicitation
Advertising as a Means of Enhancing Access
- Most common types of advertising is through brochures and Yellow Pages listings
- Law firms that provide individualized services to wealthy or corporate clients rarely
use the forms of lawyer advertising that are criticizedbillboards, newspaper ads,
radio spots and television commercials
- Lawyers in the corporate sector of practice, however, do use a variety of techniques
to bring themselves to the attention of potential clients
o Advertising is an efficient means to market legal services that are capable of
standardization but not those that require highly individualized treatment
Model Rules on Lawyer Advertising
Model Rule 7.2(a): states the general rule that advertising is permissible, subject to certain
restrictions
- Among these restrictions are that advertisements must include the name and office
address of at least one lawyer or law firm responsible for its content, M.R. 7.2(c)
Model Rule 7.1: advertising must be truthful and not misleading
- If running an add, this will still apply
Model Rule 7.3 imposes additional requirements on solicitation
Model Rule 7.4 addresses the substantive content of advertising, imposing restrictions on
what lawyers may say about their fields of practice and specialization

89

Model Rule 7.5 regulates firm names and letterheads


Model Rule 7.6 addresses lawyers who seek to get government-related business by making
political contributions, commonly known as pay to play
First Amendment Protection of Lawyer Advertising
- The First Amendment provides broad protection to lawyer efforts to gain business by
various forms of (truthful) advertising and (non-coercive) solicitation, with the
exception of in-person solicitation of legal business
- Standards are set out in Central Hudson Gas & Electric Corp. v. Public Service
Commission
o This test protects commercial speech (defined as expression solely related to
the economic interests of the speaker and its audience) that concerns lawful
activity
o Regulation is upheld only if:
(1) the asserted government interest is substantial
(2) the regulation directly advances that interest and
(3) the regulation is no more extensive than necessary to serve that
interest
Personalized Letters to persons thought to have legal problems:
Shapero v. Kentucky Bar Association
Personalized letters to persons thought to have legal problems
Facts
Petitioner, attorney Shapero, formally requested the approval of Respondent, the
Kentucky Bar Association, for a letter that he intended to mail to a select list of
people in Kentucky who had recently had their homes foreclosed upon.
Respondent conceded that the letter was not false or misleading, but declined to
approve his request on the grounds that it violated an existing state ethics rule.
Respondent denied this request, and Petitioner now appeals from the Kentucky
Supreme Courts decision in which it determined that ABA Rule 7.3 should
replace the previous governing rule.
Issue
Whether a state may, consistent with the First and Fourteenth Amendments,
categorically prohibit lawyers from soliciting legal business for pecuniary gain by
sending truthful and non-deceptive letters to potential clients known to face
particular legal problems
Holding
No. States should be able to craft far more narrow bans on certain kinds of
potentially-misleading lawyer advertising than the rule being challenged here.
Reasoni
- The court disapproved petitioners proposed letter solely
because it targeted only persons who were known to need the
ng
legal services offered in his letter, rather than the broader group
of persons so situated that they might in general find such
services useful
- Court concluded that the States blanket ban on all targeted, direct-mail
solicitation was permissible because of the serious potential for abuse
inherent in direct solicitation by lawyers of potential clients known to
need specific legal services
- Like print advertising, petitioners letter and targeted, direct-mail
solicitation generally poses much less risk of over-reaching or undue
influence than does in-person solicitation
- No matter what font/print is used, a letter does not compare to a lawyer
engaging in face-to-face solicitation

90

Written Solicitation of Accident Victims:


Florida Bar v. Went For It, Inc.
Written Solicitation of Accident victims
Facts
Florida Supreme Court adopted two amendments to its Rules of Professional
Conduct
(1) Prohibiting direct mail contact from a lawyer to a victim or the family of
the victim within thirty days of an accident or disaster
(2) Prohibiting a lawyer from accepting referrals from a service whose
actions would violate the first rule if the lawyer had performed the
activity
Reasoni
- Florida was protecting the personal privacy and tranquility of its citizens
from commercial intrusion by attorneys upon their personal grief in times
ng
of trauma.
The Internet and Other Unresolved Questions
- M.R. 7.2(a) and M.R. 7.3(a), (b), & (c) make explicit references to electronic
communication and real-time electronic contact
- M.R. 8.4(a) forbids a lawyer from violating a professional rule through the acts of
another
Free Speech Rights of Lawyers
Gentile v. State Bar of Nevada
Facts
Petitioner was disciplined for making statements to the press about a pending
case in which he represented a criminal defendant.
Petitioners client was the subject of a high publicized case and the day after
indictment of client, petitioner held press conference that stated the innocence
of his client and led the public to believe that a specific detective was in fact at
fault. Trial commenced 6 months later, and although jury was brought together
that had not been affected by the media, client was acquitted on all charges.
Reasoni
- Purpose of the press conference was to counter public opinion which he
perceived as adverse to his client, to fight back against perceived efforts
ng
of the prosecution to poison the prospective juror pool, and to publicly
present his clients side of the case
- Board found that in light of the statements, their timing, and petitioners
purpose, petitioner knew or should have known that there was a
substantial likelihood that the statements would materially prejudice the
trial Nevada Supreme Court affirmed this
Petitioners argument:
- There must be a clear and present danger of actual prejudice or an
imminent threat before any discipline may be imposed on a lawyer who
initiates a press conference
Commen
- The regulation of attorneys speech is limitedit applies only to
speech that is substantially likely to have a materially prejudicial
ts
effect
-

The dissent in this case disliked the phrase about legal precedent
o Instead should look for a clear and present danger
o 1.7
o 3.6
o 8.4d

91

Rule 3.6
- (a) A lawyer who is participating or has participated in the investigation
or litigation of a matter shall not make an extrajudicial statement that the
lawyer knows or reasonably should know will be disseminated by means
of public communication and will have a substantial likelihood of
materially prejudicing an adjudicative proceeding in the matter
-

The lawyer in the discipline case making a statement to a person who


he knows has the means and likelihood of disseminating it through the
means of public communications

This rule is constitutional a lawyer cannot make statements in a radio


program directed toward the court of appeals

Another case lawyers for a plaintiff that brought a sexual harassment


employment discrimination claim against Ford
As part of her claim she was seeking to admit evidence of prior bad acts
of her supervisors
A motion was filed prior to the trial the court denied this motion, saying
she could not use the prior bad conduct
In response to this, Ps attorneys began a campaign in the media to have
her talk about the supervisor and all of his bad acts through press
interviews, concerts, etc.
The press releases began about a month and a half before the jury was
expected to be chosen
o The press release detailed all of the prior conduct

Criticism of Particular Judges


Model Rule 8.2(a) prohibits a lawyer from making a statement that the lawyer knows to be
false or with reckless disregard as to its truth or falsity concerning the qualifications or
integrity of judicial officers and candidates for such office.
CHAPTER 11 THE STRUCTURE OF LEGAL PRACTICE
Character and Fitness
- In addition to educational requirements and passing a written bar exam, admission to
the bar requires fulfillment of a character and fitness requirement
- Following matters are considered significant enough to merit more intensive inquiry,
although they are not used as automatic exclusion:
o Criminal record, drug or alcohol abuse, repeated traffic offenses, dishonesty in
business transactions, financial difficulties, plagiarism and other cheating in
school, unauthorized practice of law, psychiatric treatment and nondisclosure
or false statements on the bar admission questionnaire
In re Hale
Facts
Respondent, Matthew Hale, was a well-known white supremacist in Illinois who
headed the World Church of the Creator. This church was solely dedicated to
the survival, expansion, and advancement of the White Race [sic] exclusively

92

Issue
Holding

Ratio
Reasoni
ng

Commen
ts

and actively preached hatred against Jews, Muslims, and nearly every other
racial, religious, and ethnic group. Although it claimed to advocate non-violence,
several of its members were involved in notorious hate crimes throughout its
existence. Respondent attended the University of Illinois Law School and passed
the Illinois bar exam in 1998. His character and fitness to serve as an attorney in
Illinois was called into question by his beliefs, however, and they are examined
here. Although Hale said he would support the US Constitution and the
Constitution of the State of Illinois in good conscious, he said it was his first
amendment right to speak up about his beliefs regarding racism.
Does Respondent have the moral character and fitness to practice law in
Illinois?
No. The board found that Hales extremist beliefs represented a gross
deficiency in moral character that left him unsuitable to practice law in Illinois.
His commitment to extreme racial hatred and discrimination would put him on a
collision course with the states Rules of Professional Conduct.
- It is not unconstitutional
-

In his challenge to this decision, Hale argued that the states


discrimination against his private beliefs should not have prevented
him from obtaining his law license. The wisdom of the boards decision
appears to have been confirmed, however, when one of Hales followers
immediately engaged in a three-day shooting spree apparently motivated
by the boards decision in this case.
Rule 4.1 of the applicable rules places the burden on the applicant to
prove by clear and convincing evidence that he has the requisite
character and fitness for admission to the practice of law -- State must
demonstrate that a legitimate state interest is sought to be protected

In re Himmel
Facts
Motorcycle accident victim Tammy Forsbergs prior attorney, John Casey, had
wrongfully withheld a portion of her settlement check. She retained Respondent,
James Himmel, to recover the remaining amount. Respondent did so by reaching
a negotiated agreement with Casey, which he later breached. Respondent then
brought suit to enforce the agreement and eventually recovered so little that he
did not receive a fee for his work. Casey was later disbarred for an unrelated
matter, and a complaint was filed against Respondent for failing to disclose
Caseys misconduct in the Forsberg case.
Issue
May a lawyer be disclipined solely for failing to report the misconduct of another
lawyer?
Holding
Yes. Respondents choice to settle rather than report Caseys misconduct was
ill-advised and he should be held responsible for it. One-year suspension.
Reasoni
This is the first case in which a lawyer was disciplined solely for failing to report
ng
the misconduct of another attorney. While this may seem unusual enough on its
face, it is further complicated by the fact that the settlement was actually the
best thing for his client-and was actually instigated by her request-and that
reporting might have actually interfered with her ability to recover from Casey.
Racist and Sexist Conduct

93

Covered by M.R. 4.3 (harassment) and M.R. 8.4(d) (conduct prejudicial to the
administration of justice) comment 3 added to 8.4 (applies to racism in the course
of representing a client)

Obligation to Report Professional Misconduct of Other Lawyers


Model Rule 8.3(a) provides: a lawyer who knows that another lawyer has committed a
violation of the Rules of Professional Conduct that raises a substantial question as to that
lawyers honesty, trustworthiness or fitness as a lawyer in other respects, shall inform the
appropriate professional authority
- Includes 2 important limitations
(1) like other important ethics rules, it requires actual knowledge of a violation
(2) it does not require reporting of all violations, but only substantial ones that raise
questions about the misbehaving lawyers fitness as a lawyer
- A third important limitation is in M.R. 8.3(c) which states that M.R. 8.3 does not
require disclosure of information otherwise protected by Rule 1.6 or information
gained by a lawyer or judge while participating in a ___ program
-

RULE
o
o
o

8.3(a) WILL BE ON EXAM


Actual knowledge
That raises a substantial question
As to the lawyers fitness, etc.

Gambro
o
o
o
o
o
o
o
o

An Illinois case
Whistleblowing
Lawyer worked for the EPA he learned of the misconduct within the EPA
The EPA became aware of certain seepages into the ground
The whistleblower became aware of this he was a lawyer within the EPA he
tried to disclose this and get damages for it
The court rules that he could not do so because he was a lawyer
He had a duty to report misconduct by the other EPA lawyers under 8.3(a)
Because he had this duty to report, the whistleblower statute is made for
people who would not otherwise whistle blow to do so, because he was
required to do so anyway, he was not able to get monetary damages for doing
so

94

Hypotheticals
1. C is a long-time client, they deliver a case of expensive wine to your house each Canada
Day. Is this permissible? And where do we go if any limitations apply? (remember it isnt
a fee, it is a gift; so what are the limitations on gifts?)
- 1.8(c) shall not solicit any substantial gift from a client, including a testamentary gift.
- As it is unsolicited it is okay.
2. A Corp. was in a hostile takeover that has taken over 3 months, they celebrate the
victorious end of the takeover for a lavish all-expense paid trip to a Caribbean resort in
thanks for those who were on the legal team and their spouses. (There was no
solicitation by the lawyers).
- Not an ethics issue, as it was not solicitedeven though the spouse benefits.
3. A Corp. asks you to prepare a gift instrument of 10 shares of A Corp. to each of the
lawyers working on the takeover described above which you were a part. No solicitation.
- You cannot solicit or prepare the gift on behalf of the client, and so as the lawyers are
not related to the client this cannot be done if it is a substantial gift. So it depends on
if this is substantial or notneed to know the value of the shares then to answer this
question.
- If in fact 10 shares represents a significant interest in the corporation then there is a
problem, but if it doesnt represent a lot, then it is not a problem
- The general standards of fairness are what really have to be met
4. We all worked on the matter for the client, and B did not. If they asked B to prepare the
10 shares (and B will not receive them) is that okay?
- Assuming this is a lot (substantial gift), it is not allowed too because of fears of
overreaching and influencing the client wronglyas the action is still within the firm.
- Imputation1.10 (the firm)
o No ethics rules that discipline a firm, only lawyers. But sometimes that lawyers
discipline can be traced through the firm association.
5. You are a young associate in a law firm and one of the partners comes to you and gives
you this task: a client who is facing the potential of federal fraud and banking related
charges has asked for a list of the countries that would be most favorable to her if she
did not want to be extradited back to the United States. Your partner gives you three
nations: Chile, Venezuela and Bolivia. And tells you to research which of these three
would be the safest for your client to go. Is this a problem for you as a lawyer? And if so,
why and how?
- Yes it is a problem. Although you are not directly helping them to commit a crime,
you are providing information that would help them in committing a crime indirectly
1.2(d) as you are giving them the information to encourage the process to commit a
crime
- You will be seen as an aider and abetter if the purpose is to run away with the money,
the chances are you will be liable, and seen as a part of the conspiracy to defraud the
US and others
6. There is a group of protestors that are protesting manufacturers of parts for missiles of
nuclear warfare, and they have been charged with trespassing as civil disobedience
multiple times in the past. What if the group comes to you before their next outing and
says their plan is to trespass and get arrested in order to make their point. They want
you to represent them at the hearing and basically give you the bail money and tell you
to hold it for them until they get arrested and then pay their bail money and represent
them.
- This is the same idea as above. You cannot do this as per Rule 1.2(c).
7. You are a lawyer who specializes in estate planning. You are aware that under Medicaid
now if you have over a certain amount of money you have to pay that money as part of

95

your care if you end up in a nursing home. Until you deplete your savings below a certain
amount, you pay. You are a lawyer and the client is about to go into the home and they
have a million dollars, and come to you for advice, and you say they can spend down in
their savings to get to that line so the government does not get it for Medicaid.
Purchases that are legal exemptionsex. caskets.
- This is allowed. Originally these were violations as they ultimately were to evade the
payment. Then it was determined to be lawyering that was good advice. Careful
about how much you know about why they wanted the info. Although it results in
harm to the US it is considered to be good lawyering.
8. Seller has contracted with Buyer to deliver 1000 widgets per month for an agreed upon
price. Because of dramatic and unanticipated changes in the cost of materials, Seller
faces enormous losses. Seller consults Lawyer who advises Seller that Seller is bound by
contract but that in this circumstance it will be cheaper for Seller to pay contract
damages than to fulfill the contract. Seller breaches contract. Was Lawyer's conduct
proper? Did Lawyer violate any MRPC?
- Does the seller now have a cause of action against the lawyer?
o Seller is not getting the benefit of the bargain
o The lawyer facilitates the clients conductas it was his direct advice to
breach
o Lawyer is in violation tells client to intentionally breach the contract
o This is an intentional violation of a civil obligation, however
Hazard: Other than failure to perform a contract, or failure to
sustain a good faith claim to property; these are not the types
of claims that lawyers are held responsible for
- What if instead of a contract circumstance it was in a property circumstances?
Legitimate claim between two neighbours and both of them claim Blackacre. The one
approaches the lawyer asking for advice, and the reply is that there is a 70% chance
that a claim would not succeed. The client then takes possession of the property, and
invites the lawyer over. Trees are cut down by the client. The other neighbour then
files a claim and wins.
9. Lawyer is outside counsel for company that manufactures truck bodies. The company's
truck body design does not include a bumper or bar that would prevent cars from going
under the rear of the truck body. The bumper or bar would cost about $400 per truck
body but would prevent hundreds of fatalities each year. A California trial court has
already upheld a jury verdict imposing liability on the grounds that the absence of a
preventative device constitutes a design defect. The company anticipates further
litigation of the matter and expects to lose some of the cases, but believes the cost of
the vigorous defense and some losses will be lower than the cost of including the
preventative device, which is not currently required by the federal safety regulations.
Lawyer reviews the situation with company officials and agrees to defend it in lawsuits as
they arise. Is Lawyer's conduct proper? Did Lawyer's conduct violate any MRPC?
- The lawyer hasnt done anything to actually undermine the laws; theres no statutory
requirement for the company to install this device so the lawyer is simply ensuring
that no further lawsuits are brought to the company. No real issue on the ethics side.
10. Lawyer represents a tampon manufacturer in a toxic shock syndrome case. During the
course of the litigation brought by a woman who is severely harmed by an infection
caused by the tampon, Lawyer engages in the following conduct: (1) At the pretrial
depositions Lawyer questions the Plaintiff at great length and in great detail about her
sexual behavior before and during marriage; (2) at trial, Lawyer seeks to cross examine
the Plaintiff about the same matters, and, after the trial judge rules that the identity
of sexual partners is irrelevant, Lawyer loudly and falsely says, in the hearing of the jury,
"But, your honor, this woman has been sleeping around with scores of men;" (3) during
closing argument Lawyer refers to the "promiscuity" and "bad moral character" of the
Plaintiff.
- There is a litigation immunity/privilege that lawyers and parties alike enjoy

96

There are some rare and very extreme examples where that privilege does not cover
you as a lawyer, but for the most part, the lawyer is going to be protected by that
privilege
Anything that is said outside of the court also has some protection available
RULE 4.4(a), 8.4(c), 3.5
Can still be disciplined for violating the rules, as the privilege is to protect against
civil action only
o 8.4(a), 3.4(e) no evidence in the record of promiscuity as the questions were
not answered
o 4.4(a) while you could argue that discovery is broad and that there was a
theory in place with these questions, you are still answerable to the rules for
this behaviour
o 8.4(c), 3.5(d) would almost be applicable (falsely stated)[Michigans rule is
much more broad and is for discourteous conduct towards the tribunal]

97

Potrebbero piacerti anche